F, Mock2

You might also like

Download as pdf or txt
Download as pdf or txt
You are on page 1of 62

Practice Mock Exam - 73%

Question 1
[4062318] Of the following, which is not one of the key environmental concerns in the investment community?
A
Climate change.

B
Natural resources.

C
Carbon pricing.

D
Pollution and waste.
You answered : C - Carbon pricing.
The correct answer is: C - Carbon pricing.
Explanation
The key environmental concerns within the investment community are currently considered to be climate change, natural
resources, and pollution and waste. Carbon pricing is an approach to dealing with climate change.
Reference: Chapter 3 section 1

Question 2
[4062319] Which of the following statements is true regarding the alignment of sovereign wealth funds with ESG concerns?
A
All sovereign wealth funds are aligned with ESG concerns.

B
Most sovereign wealth funds are aligned with ESG concerns.

C
Some sovereign wealth funds are aligned with ESG concerns.

D
No sovereign wealth funds are aligned with ESG concerns.
You answered : C - Some sovereign wealth funds are aligned with ESG concerns.
The correct answer is: C - Some sovereign wealth funds are aligned with ESG concerns.
Explanation
Some sovereign wealth funds are aligned with ESG concerns, although the lack of transparency makes assessment difficult.
Reference: Chapter 2 section 3

Question 3
[4062320] Which of the following statements is true regarding the correlation of the ESG ratings provided by third-party ESG
data providers?
A
The ratings are almost perfectly negatively correlated.

B
The ratings are almost perfectly positively correlated.

C
The ratings are negatively correlated but vary significantly.

D
The ratings are positively correlated but vary significantly.
You answered : D - The ratings are positively correlated but vary significantly.
The correct answer is: D - The ratings are positively correlated but vary significantly.
Explanation
The ratings provided by third-party ESG data providers are positively correlated but vary significantly.
Reference: Chapter 7 section 3

Question 4
[4062321] Which of the following is not a quantitative factor that will be considered by credit rating agency when assessing
corporate credit risk?
A
The strength of the issuer's balance sheet.

B
The extent of the issuer's human rights violations.

C
The issuer's overall bankruptcy risk .

D
How the issuer compares to other issuers.
You answered : B - The extent of the issuer's human rights violations.
The correct answer is: B - The extent of the issuer's human rights violations.
Explanation
The extent of the issuer's human rights violations would not be considered a quantitative factor.
Reference: Chapter 7 section 8

Question 5
[4062322] Within the IIRC Framework for evaluating different forms of capital, which of the following is NOT a form of capital?
A
Intellectual capital.

B
Management capital.

C
Manufactured capital.

D
Human capital.
You answered : B - Management capital.
The correct answer is: B - Management capital.
Explanation
The following are identified within the IIRC Framework for evaluating different forms of capital: financial capital; manufactured
capital; intellectual capital; human capital; social and relationship capital; natural capital.
Reference: Chapter 7 section 5

Question 6
[4062323] What may be described as individuals' competencies, capabilities and experiences, and their motivations to
innovate?
A
Intellectual capital.

B
Natural capital

C
Human capital.

D
Social capital.
You answered : C - Human capital.
The correct answer is: C - Human capital.
Explanation
Human capital may be described as individuals' competencies, capabilities and experiences, and their motivations to innovate.
Reference: Chapter 7 section 3

Question 7
[4062168] Which type of fund manager is most able to provide active stewardship for investors?
A
Index tracking equity fund managers.

B
Broad bond index tracking fund managers.

C
Long duration corporate bond fund managers.

D
Fundamental active equity fund managers.
You answered : D - Fundamental active equity fund managers.
The correct answer is: D - Fundamental active equity fund managers.
Explanation
Concentrated portfolios are more easily resourced from a stewardship perspective. Those firms that pride themselves on being
active stewards are genuinely concentrated and can commit significant resources to these activities. This leads to one way of
addressing the resourcing issue: Portfolio managers themselves becoming more actively involved in stewardship. This is natural
for fundamental active equity managers who hold concentrated portfolios of stocks and would typically maintain a constant
dialogue with management. Stewardship becomes challenging with larger portfolios of many stocks where managers may not
have a direct dialogue with all the companies.
Reference: Chapter 9 section 4

Question 8
[4062169] Which of the following is not a typical method by which institutional investors reflect ESG considerations?
A
Integrating ESG into investment considerations.

B
Engaging in public policy consultations on ESG issues.

C
Acting as a whistleblower in cases of corporate management rule violations.

D
Engaging actively with the boards and management of investee companies.
You answered : B - Engaging in public policy consultations on ESG issues.
The correct answer is: C - Acting as a whistleblower in cases of corporate management rule violations.
Explanation
Institutional investors typically reflect ESG considerations in three ways, namely by: Integrating ESG into investment
considerations; engaging actively with the boards and management of investee companies, and by engaging in public policy
consultations on ESG issues.
Reference: Chapter 1 section 4

Question 9
[4062170]

Several questions are associated with the following case study. The material given in the case study will not change.

You are a senior fund manager for a UK pension management group, the group being signatories to the UN Global Compact

Principles and the Principles for Responsible Investment (PRI).

The group manage a number of multi-asset pension funds, most primarily investing in a combination of shares and bonds which

are weighted to most appropriately match the liabilities of the relevant pension scheme.

To fulfil their obligations as PRI signatories the group wish to develop their systems in order to integrate EGS factors into the

investment decision making and portfolio management processes. The group have particular concerns regarding governance

factors as they are assessed as representing the most significant risk to the group’s funds based on the investments they hold.

In addition to satisfying your PRI obligations, you are looking to use this process to reduce the downside risk faced by the

group’s funds; again, particularly in respect of governance concerns. In doing so, however, you must ensure that your approach

does not reduce the investment universe available to the funds as that is not in your investment remit.
As a senior fund manager you have been co-opted into the ESG integration development team, and there are several

alternative ESG integration approaches that are being considered, there having been much discussion on the most appropriate

approach.

Although the precise approach to integration is still to be resolved, the development team have agreed that the group must be

more pro-active in its engagement efforts, either individually or collectively with other fund management groups. With this in

mind, it is your intention to engage with a number of investee companies on various ESG issues in order to get the ball rolling.

The ESG data that you are assessing with respect to these two issues comes in a variety of formats, some being qualitative and

some being quantitative, and you are devising a technique for incorporating any relevant data into your corporate analysis.

Given the identified concerns that are driving your engagement, and in compliance with Principle 11 of the UK Stewardship

Code, who should you engage with first and what escalation procedures should you follow if necessary?

A
The CEO or CFO, with escalation to the non-executive directors.

B
The non-executive directors, with escalation to the chair.

C
The chair, with escalation to the senior independent director or to other non-executives.

D
The company secretary with escalation to the chair. .
You answered : C - The chair, with escalation to the senior independent director or to other non-executives.
The correct answer is: C - The chair, with escalation to the senior independent director or to other non-executives.
Explanation
For governance matters, the usual starting point is the chair, with escalation to the senior independent director or to other non-
executives. See Chapter 6 Section 5
Reference: Chapter 6 Section 5

Question 10
[4062171]
Several questions are associated with the following case study. The material given in the case study will not change.

You are a senior fund manager for a UK pension management group, the group being signatories to the UN Global Compact

Principles and the Principles for Responsible Investment (PRI).The group manage a number of multi-asset pension funds, most

primarily investing in a combination of shares and bonds which are weighted to most appropriately match the liabilities of the

relevant pension scheme.To fulfil their obligations as PRI signatories the group wish to develop their systems in order to

integrate EGS factors into the investment decision making and portfolio management processes. The group have particular

concerns regarding governance factors as they are assessed as representing the most significant risk to the group’s funds

based on the investments they hold.In addition to satisfying your PRI obligations, you are looking to use this process to reduce

the downside risk faced by the group’s funds; again, particularly in respect of governance concerns. In doing so, however, you

must ensure that your approach does not reduce the investment universe available to the funds as that is not in your investment

remit.As a senior fund manager you have been co-opted into the ESG integration development team, and there are several

alternative ESG integration approaches that are being considered, there having been much discussion on the most appropriate

approach.Although the precise approach to integration is still to be resolved, the development team have agreed that the group

must be more pro-active in its engagement efforts, either individually or collectively with other fund management groups. With

this in mind, it is your intention to engage with a number of investee companies on various ESG issues in order to get the ball

rolling.The ESG data that you are assessing with respect to these two issues comes in a variety of formats, some being
qualitative and some being quantitative, and you are devising a technique for incorporating any relevant data into your corporate

analysis.Which of the following comments would be most accurate in respect of your desire to reduce the downside risks faced

by the group's funds?


A
ESG engagement has little impact on downside risk but has a positive impact on returns through identifying opportunities, hence
results in positive outcomes. The effects are strongest in relation to governance, then for environmental issues.

B
ESG engagement leads to a reduction in downside risk, and the effect is stronger the more successful the engagement is. The
effects are strongest in relation to governance, then for social issues.

C
ESG engagement can lead to significant improvements in corporate governance behaviour and in corporate risk in general. The
effects are strongest for environmental issues and then governance.

D
ESG engagement has little impact on corporate governance behaviour but may lead to substantial improvements in respect of
environmental and social issues, hence is a positive activity. The effects are strongest for environmental issues and then social
issues
You answered : B - ESG engagement leads to a reduction in downside risk, and the effect is stronger the more successful
the engagement is. The effects are strongest in relation to governance, then for social issues.
The correct answer is: B - ESG engagement leads to a reduction in downside risk, and the effect is stronger the more
successful the engagement is. The effects are strongest in relation to governance, then for social issues.
Explanation
ESG engagement leads to a reduction in downside risk, and the effect is stronger the more successful the engagement is. The
effects are strongest in relation to governance, then for social issues (so long as these are also associated with work on
governance). See Chapter 6 Section 1
Reference: Chapter 6 section 1

Question 11
[4062172]

Several questions are associated with the following case study. The material given in the case study will not change.
You are a senior fund manager for a UK pension management group, the group being signatories to the UN

Global Compact Principles and the Principles for Responsible Investment (PRI).

The group manage a number of multi-asset pension funds, most primarily investing in a combination of shares and bonds which

are weighted to most appropriately match the liabilities of the relevant pension scheme.To fulfil their obligations as PRI
signatories the group wish to develop their systems in order to integrate EGS factors into the investment decision making and

portfolio management processes. The group have particular concerns regarding governance factors as they are assessed as

representing the most significant risk to the group’s funds based on the investments they hold.In addition to satisfying your PRI

obligations, you are looking to use this process to reduce the downside risk faced by the group’s funds; again, particularly in

respect of governance concerns. In doing so, however, you must ensure that your approach does not reduce the investment

universe available to the funds as that is not in your investment remit.As a senior fund manager you have been co-opted into the

ESG integration development team, and there are several alternative ESG integration approaches that are being considered,

there having been much discussion on the most appropriate approach.Although the precise approach to integration is still to be

resolved, the development team have agreed that the group must be more pro-active in its engagement efforts, either

individually or collectively with other fund management groups. With this in mind, it is your intention to engage with a number of

investee companies on various ESG issues in order to get the ball rolling.The ESG data that you are assessing with respect to

these two issues comes in a variety of formats, some being qualitative and some being quantitative, and you are devising a

technique for incorporating any relevant data into your corporate analysis.

You are considering engaging with your investee companies in collaboration with other fund managers and of using the services

of proxy voting providers. Which of the following statements are valid in this context?
i. Collective engagement can take place through industry initiatives and collaboration platforms, such as one offered by the

Principles for Responsible Investment (PRI) or through the Investor Forum in the UK.

ii. Almost all institutional investors lean at least in part on one group of these service providers, the proxy voting advisory firms

who offer analysis and (in most cases) voting recommendations across all public companies.

iii. Engagement styles vary depending on the heritage of stewardship teams, and there is a distinction in mindset and approach

between those teams with a history of governance-led engagement and those that have worked more on the environmental and

social side.

iv. Active investors typically start with an issue, whether identified by the team from news or broader analysis, or through a

screening or other research provider, and seek to engage with companies impacted by that issue.

A
i and ii only.

B
ii and iii only.

C
i, ii and iii only.

D
i, ii, iii and iv.
You answered : C - i, ii and iii only.
The correct answer is: C - i, ii and iii only.
Explanation
Passive investors typically start with an issue, whether identified by the team from news or broader analysis, or through a
screening or other research provider, and seek to engage with companies impacted by that issue. Active investors start with the
company itself and its business issues and develop a tailored engagement approach cutting across a range of issues, often with
the investment teams taking a leading role. Companies selected for this approach are often identified from investment
underperformers or ones that trigger other financial or ESG metrics. See Chapter 6 Section 4
Reference: Chapter 6 section 4

Question 12
[4062173]

Several questions are associated with the following case study. The material given in the case study will not change.

You are a senior fund manager for a UK pension management group, the group being signatories to the UN Global Compact

Principles and the Principles for Responsible Investment (PRI).The group manage a number of multi-asset pension funds, most

primarily investing in a combination of shares and bonds which are weighted to most appropriately match the liabilities of the

relevant pension scheme.To fulfil their obligations as PRI signatories the group wish to develop their systems in order to

integrate EGS factors into the investment decision making and portfolio management processes. The group have particular

concerns regarding governance factors as they are assessed as representing the most significant risk to the group’s funds

based on the investments they hold.In addition to satisfying your PRI obligations, you are looking to use this process to reduce

the downside risk faced by the group’s funds; again, particularly in respect of governance concerns. In doing so, however, you

must ensure that your approach does not reduce the investment universe available to the funds as that is not in your investment

remit.As a senior fund manager you have been co-opted into the ESG integration development team, and there are several

alternative ESG integration approaches that are being considered, there having been much discussion on the most appropriate

approach.Although the precise approach to integration is still to be resolved, the development team have agreed that the group

must be more pro-active in its engagement efforts, either individually or collectively with other fund management groups. With
this in mind, it is your intention to engage with a number of investee companies on various ESG issues in order to get the ball

rolling.The ESG data that you are assessing with respect to these two issues comes in a variety of formats, some being

qualitative and some being quantitative, and you are devising a technique for incorporating any relevant data into your corporate

analysis.

You would like your engagement with the company to be both positive and effective and are, accordingly, preparing for your

pre-arranged meeting. When discussing the various relevant factors with your fellow fund managers, which of the following

would be the most valid comment to make?

A
Governance is a stewardship issue, hence may not be handled through engagement which may only cover environmental and
social issues.

B
Engagement can only be effectively achieved through exercising voting powers at the company's AGM, however engagement
meetings allow you to inform the company of how you intend to vote.

C
Engagement may encompass any issue that affect the long-term value of the business, evidence showing that it adds value
through reducing risks or taking advantage of opportunities.

D
Identifying and discussing governance concerns is a relatively new phenomenon and there is little evidence regarding its
effectiveness or its ability to achieve the desired change.
You answered : C - Engagement may encompass any issue that affect the long-term value of the business, evidence
showing that it adds value through reducing risks or taking advantage of opportunities.
The correct answer is: C - Engagement may encompass any issue that affect the long-term value of the business, evidence
showing that it adds value through reducing risks or taking advantage of opportunities.
Explanation
Engagement may encompass any issue that affect the long-term value of the business, evidence showing that it adds value
through reducing risks or taking advantage of opportunities. See Chapter 6 section 1
Reference: Chapter 6 section 1

Question 13
[4062174]

Several questions are associated with the following case study. The material given in the case study will not change.

You are a senior fund manager for a UK pension management group, the group being signatories to the UN Global Compact

Principles and the Principles for Responsible Investment (PRI).The group manage a number of multi-asset pension funds, most

primarily investing in a combination of shares and bonds which are weighted to most appropriately match the liabilities of the

relevant pension scheme.To fulfil their obligations as PRI signatories the group wish to develop their systems in order to

integrate EGS factors into the investment decision making and portfolio management processes. The group have particular

concerns regarding governance factors as they are assessed as representing the most significant risk to the group’s funds

based on the investments they hold.In addition to satisfying your PRI obligations, you are looking to use this process to reduce

the downside risk faced by the group’s funds; again, particularly in respect of governance concerns. In doing so, however, you

must ensure that your approach does not reduce the investment universe available to the funds as that is not in your investment

remit.As a senior fund manager you have been co-opted into the ESG integration development team, and there are several

alternative ESG integration approaches that are being considered, there having been much discussion on the most appropriate

approach.Although the precise approach to integration is still to be resolved, the development team have agreed that the group
must be more pro-active in its engagement efforts, either individually or collectively with other fund management groups. With

this in mind, it is your intention to engage with a number of investee companies on various ESG issues in order to get the ball

rolling.The ESG data that you are assessing with respect to these two issues comes in a variety of formats, some being

qualitative and some being quantitative, and you are devising a technique for incorporating any relevant data into your corporate

analysis.

Following your engagement with one investee company you have determined that the company is willing to proactively inform
investors when engagement objectives and targets have been met. How would you characterize this response?

A
A corporate factor that represents an enabler from an investor perspective.

B
A relationship factor that represents a barrier from a corporate perspective.

C
An investor factor that represents an enabler from a corporate perspective.

D
A corporate factor that represents an enabler from a corporate perspective.
You answered : A - A corporate factor that represents an enabler from an investor perspective.
The correct answer is: A - A corporate factor that represents an enabler from an investor perspective.
Explanation
The willingness of the company to proactively inform investors when engagement objectives and targets have been met is a
corporate factor that represents an enabler from an investor perspective. See Chapter 6 Section 5
Reference: Chapter 6 section 5

Question 14
[4062175] According to the Pensions and Lifetime Savings Association, there are various disclosures that may be considered
when determining whether a fund manager has effectively integrated ESG factors into their investment processes. Which of the
following is not one of those disclosures?
A
Examples of where and why the manager is prepared to take either stock or sector ESG risks or where it sees opportunities.

B
Quantitative or qualitative examples of material ESG factors identified in fundamental analysis and stock valuation.

C
Only quantitative examples of material ESG factors identified in fundamental analysis and stock valuation.

D
Identification of long-term ESG secular trends and themes and the extent to which they have influenced portfolio construction
decisions.
You answered : C - Only quantitative examples of material ESG factors identified in fundamental analysis and stock
valuation.
The correct answer is: C - Only quantitative examples of material ESG factors identified in fundamental analysis and stock
valuation.
Explanation
The UK’s Pensions and Lifetime Savings Association (PLSA) published a disclosure guide setting out some pared down
expectations for managers reporting on both ESG integration and stewardship activities. The disclosure on ESG integration
asks for separate disclosure on both (1) identification of ESG risk and (2) the management and monitoring of ESG risks and
opportunities, with suggested possible disclosures in respect of each. The PLSA allows a number of ways to assess if the fund
manager has integrated the ESG factors effectively. These are as follows: (1) examples of where and why the manager is
prepared to take either stock or sector ESG risks or where it sees opportunities; (2) quantitative or qualitative examples of
material ESG factors identified in fundamental analysis and stock valuation; and (3) identification of long-term ESG secular
trends and themes (as potential determinants of future growth/valuation etc.) and the extent to which they have influenced
portfolio construction decisions. Reference Chapter 9 section 4
Reference: Chapter 9 section 7

Question 15
[4062176] The adoption of the sustainable development goals (SDGs) in 2015 was a strong signal that one of the major global
challenges of the twenty-first century will be how to mitigate against the adverse social impact of environmental factors. What is
the first of these sustainable development goals (SDG 1)?
A
To ensure universal good health and well-being for all ages.

B
To end poverty in all its forms everywhere.

C
To end hunger, achieve food security and improved nutrition and promote sustainable agriculture.

D
To take urgent action to combat climate change and its impacts.
You answered : B - To end poverty in all its forms everywhere.
The correct answer is: B - To end poverty in all its forms everywhere.
Explanation
SDG 1 goal is to end poverty in all its forms everywhere.
Reference: Chapter 1 section 5

Question 16
[4062177] There are six United Nations Principles for Responsible Investment (PRI), which of them expects signatories to
incorporate ESG into their investment approach?
A
Principle 1.

B
Principle 2.

C
Principle 3.

D
Principle 4.
You answered : C - Principle 3.
The correct answer is: A - Principle 1.
Explanation
There are six Principles for Responsible Investment: 1. We will incorporate ESG issues into investment analysis and decision-
making processes; 2. We will be active owners and incorporate ESG issues into our ownership policies and practices, 3. We will
seek appropriate disclosure on ESG issues by the entities in which we invest; 4. We will promote acceptance and
implementation of the Principles within the investment industry; 5. We will work together to enhance our effectiveness in
implementing the Principles; 6. We will each report on our activities and progress towards implementing the Principles.
Reference: Chapter 1 section 5

Question 17
[4062178] Which report examined the economics of climate change to understand the nature of the economic challenges and
how they can be met, and was of particular influence on the investment industry?
A
The Brundtland Report.

B
The Stern Report.

C
The Freshfields Report.

D
The Walker report.
You answered : B - The Stern Report.
The correct answer is: B - The Stern Report.
Explanation
The Stern report examined the economics of climate change to understand the nature of the economic challenges and how they
can be met, and was a particular influence on the investment industry.
Reference: Chapter 2 section 1
Question 18
[4062179] Which of the following statements is true in respect of the adoption of responsible investment?
A
The adoption of responsible investment by retail investors has been more rapid than that of institutional investors.

B
The adoption of responsible investment by retail investors has been generally slower than that of institutional investors.

C
The adoption of responsible investment by retail investors has been broadly in line with that of institutional investors.

D
The adoption of responsible investment by retail investors has varied significantly when compared to that of institutional
investors.
You answered : D - The adoption of responsible investment by retail investors has varied significantly when compared to
that of institutional investors.
The correct answer is: B - The adoption of responsible investment by retail investors has been generally slower than that of
institutional investors.
Explanation
The adoption of responsible investment by retail investors has been generally slower than that of institutional investors.
Reference: Chapter 2 section 3

Question 19
[4062180] There are a number of reasons why growth in the availability of ESG indices is important to the development of the
ESG market. Which of the following is NOT one of those reasons?
A
They provide benchmarks for ESG performance analysis.

B
They represent ESG portfolios optimised to achieve the best combination of risk and return.

C
They are the basis for exchange-traded funds (ETFs).

D
They help to set standards for categorising ESG..
You answered : D - They help to set standards for categorising ESG..
The correct answer is: B - They represent ESG portfolios optimised to achieve the best combination of risk and return.
Explanation
ESG indices are the building blocks for exchange-traded funds (ETFs), help to set standards for categorising ESG, and provide
benchmarks for performance analysis, however they do not represent optimised ESG portfolios.
Reference: Chapter 2 section 3

Question 20
[4062181] Within France, institutional investors are required to consider climate change under which article of the French Law
on Energy Transition for Green Growth?
A
Article 172.

B
Article 173.

C
Article 174.

D
Article 175.
You answered : B - Article 173.
The correct answer is: B - Article 173.
Explanation
Article 173-VI of the France’s Law on Energy Transition for Green Growth requires both fund managers and asset owners (in
France, these are primarily insurance firms) to describe how they incorporate ESG factors into their investment strategy, across
all asset classes.
Reference: Chapter 2 section 2
Question 21
[4062182]

Several questions are associated with the following case study. The material given in the case study will not change.

You have been asked to prepare a report on the various qualitative or quantitative ways in which ESG factors may be analysed

and integrated into the investment decision making process, culminating in a recommended approach. As a mathematician,

your natural preference is to adopt a quantitative approach, as this easily feeds into your valuation models. However, you are

aware that there are a number of more judgemental qualitative factors which also need to be addressed and which may be

difficult to precisely quantify.

To help with your deliberations you have obtained the following ESG attribution analysis for one of your investee companies

from Sustainalytics and have examined their approach to establishing ESG scores.

Issue Name Contributio Subindustry Company Manageable Management ESG Risk

n to ESG Exposure Exposure Risk Factor Score % Rating

Risk Rating %

Corporate governance 26.2% 9.0 9.0 100 53.1 4.2

E&S impact of products and services 17.4% 5.0 5.0 100 43.9 2.8

Human capital 14.4% 4.0 4.0 95 44.4 2.3

Resource use 11.9% 5.0 5.0 80 77.3 1.9

Carbon – own operations 7.6% 6.0 5.7 100 78.6 1.2

Human rights – supply chain 7.3% 4.0 4.2 80 90.0 1.2

Land use and biodiversity – supply 6.0% 3.0 3.5 80 90.0 1.0

chain

Occupational health and safety 5.2% 3.0 3.0 95 75.8 0.8

Resource use – supply chain 2.5% 2.0 2.0 80 100.0 0.4

Business ethics 1.5% 3.0 3.0 95 96.9 0.2

Product governance 0.0% 8.0 8.0 100 100.0 0.0

Overall 100.0% 52.0 52.4 93.5 74.1 16.1

Within this analysis the:

 Sub-industry exposure indicates the sector average exposure;


 Company exposure indicates the actual exposure of the investee company;
 Manageable risk factor represents the proportion each risk that is capable of being managed by the company;
 Management score represents the proportion of the manageable risk which is managed by the company.

How does the company preform relative to its sector (sub-industry) in respect of carbon emissions from its own

operations and supply chain human rights?


A
The company performs better than the sector average for both carbon emissions and supply chain human rights.

B
The company performs better than the sector average for carbon emissions but below the sector average for supply chain
human rights.

C
The company is below the sector average for carbon emissions but performs better than the sector average for supply chain
human rights.

D
The company is below the sector average for both carbon emissions and supply chain human rights.
You answered : B - The company performs better than the sector average for carbon emissions but below the sector
average for supply chain human rights.
The correct answer is: B - The company performs better than the sector average for carbon emissions but below the sector
average for supply chain human rights.
Explanation
The company performs better than the sector average for carbon emissions, having an exposure of 5.7 against a sub-industry
exposure of 6.0. It is, however, below the sector average for supply chain human rights, having a score of 4.2 against a sub-
industry average of 4.0. See Chapter 7 section 5
Reference: Chapter 7 section 5

Question 22
[4062183]

Several questions are associated with the following case study. The material given in the case study will not change.

You have been asked to prepare a report on the various qualitative or quantitative ways in which ESG factors may be analysed

and integrated into the investment decision making process, culminating in a recommended approach. As a mathematician,

your natural preference is to adopt a quantitative approach, as this easily feeds into your valuation models. However, you are

aware that there are a number of more judgemental qualitative factors which also need to be addressed and which may be

difficult to precisely quantify.

To help with your deliberations you have obtained the following ESG attribution analysis for one of your investee companies

from Sustainalytics and have examined their approach to establishing ESG scores.

Issue Name Contributio Subindustry Company Manageable Management ESG Risk

n to ESG Exposure Exposure Risk Factor Score % Rating

Risk Rating %

Corporate governance 26.2% 9.0 9.0 100 53.1 4.2

E&S impact of products and services 17.4% 5.0 5.0 100 43.9 2.8

Human capital 14.4% 4.0 4.0 95 44.4 2.3

Resource use 11.9% 5.0 5.0 80 77.3 1.9

Carbon – own operations 7.6% 6.0 5.7 100 78.6 1.2

Human rights – supply chain 7.3% 4.0 4.2 80 90.0 1.2

Land use and biodiversity – supply 6.0% 3.0 3.5 80 90.0 1.0

chain

Occupational health and safety 5.2% 3.0 3.0 95 75.8 0.8

Resource use – supply chain 2.5% 2.0 2.0 80 100.0 0.4

Business ethics 1.5% 3.0 3.0 95 96.9 0.2

Product governance 0.0% 8.0 8.0 100 100.0 0.0


Overall 100.0% 52.0 52.4 93.5 74.1 16.1

Manageable exposure 49.0


(93.5%)
Managed exposure 36.3
(74.1%)

Within this analysis the:

 Sub-industry exposure indicates the sector average exposure;


 Company exposure indicates the actual exposure of the investee company;
 Manageable risk factor represents the proportion each risk that is capable of being managed by the company;
 Management score represents the proportion of the manageable risk which is managed by the company.

Which of the following places the supply chain risks in the correct descending order?
A
Human rights, land use and biodiversity, resource use.

B
Land use and biodiversity, human rights, resource use.

C
Resource use, land use and biodiversity, human rights.

D
Resource use, human rights, land use and biodiversity.
You answered : A - Human rights, land use and biodiversity, resource use.
The correct answer is: A - Human rights, land use and biodiversity, resource use.
Explanation
The noted supply chain exposures of the company are: human rights 4.2, Land use and diversity 3.5, Resource use 2.0. See
Chapter 7 Section 5
Reference: Chapter 7 section 5

Question 23
[4062184]

Several questions are associated with the following case study. The material given in the case study will not change.

You have been asked to prepare a report on the various qualitative or quantitative ways in which ESG factors may be analysed

and integrated into the investment decision making process, culminating in a recommended approach. As a mathematician,

your natural preference is to adopt a quantitative approach, as this easily feeds into your valuation models. However, you are

aware that there are a number of more judgemental qualitative factors which also need to be addressed and which may be

difficult to precisely quantify.

To help with your deliberations you have obtained the following ESG attribution analysis for one of your investee companies

from Sustainalytics and have examined their approach to establishing ESG scores.

Issue Name Contribution Subindustry Company Manageable Management ESG Risk

to ESG Risk Exposure Exposure Risk Factor Score % Rating

Rating %

Corporate governance 26.2% 9.0 9.0 100 53.1 4.2


E&S impact of products and services 17.4% 5.0 5.0 100 43.9 2.8

Human capital 14.4% 4.0 4.0 95 44.4 2.3

Resource use 11.9% 5.0 5.0 80 77.3 1.9

Carbon – own operations 7.6% 6.0 5.7 100 78.6 1.2

Human rights – supply chain 7.3% 4.0 4.2 80 90.0 1.2

Land use and biodiversity – supply 6.0% 3.0 3.5 80 90.0 1.0

chain

Occupational health and safety 5.2% 3.0 3.0 95 75.8 0.8

Resource use – supply chain 2.5% 2.0 2.0 80 100.0 0.4

Business ethics 1.5% 3.0 3.0 95 96.9 0.2

Product governance 0.0% 8.0 8.0 100 100.0 0.0

Overall 100.0% 52.0 52.4 93.5 74.1 16.1


Manageable exposure 49.0
(93.5%)
Managed exposure 36.3
(74.1%)

Within this analysis the:

 Sub-industry exposure indicates the sector average exposure;


 Company exposure indicates the actual exposure of the investee company;
 Manageable risk factor represents the proportion each risk that is capable of being managed by the company;
 Management score represents the proportion of the manageable risk which is managed by the company.

Based on the company's overall exposure of 52.4 and using the other relevant overall exposure figures, what is

the overall unmanageable risk?


A
Unmanageable risk is 52.4.

B
Unmanageable risk is 49.0.

C
Unmanageable risk is 36.3.

D
Unmanageable risk is 3.4.
You answered : D - Unmanageable risk is 3.4.
The correct answer is: D - Unmanageable risk is 3.4.
Explanation
The company's total exposure is 52.4 and the manageable risk is 49.0, hence the remaining 3.4 is unmanageable. The other
options are clearly incorrect as they are the total exposure, manageable exposure and managed exposure contained within the
table.. See Chapter 7 Section 3
Reference: Chapter 7 section 3

Question 24
[4062185]

Several questions are associated with the following case study. The material given in the case study will not change.

You have been asked to prepare a report on the various qualitative or quantitative ways in which ESG factors may be analysed

and integrated into the investment decision making process, culminating in a recommended approach. As a mathematician,

your natural preference is to adopt a quantitative approach, as this easily feeds into your valuation models. However, you are
aware that there are a number of more judgemental qualitative factors which also need to be addressed and which may be

difficult to precisely quantify.

To help with your deliberations you have obtained the following ESG attribution analysis for one of your investee companies

from Sustainalytics and have examined their approach to establishing ESG scores.

Issue Name Contributio Subindustry Company Manageable Management ESG Risk

n to ESG Exposure Exposure Risk Factor Score % Rating

Risk Rating %

Corporate governance 26.2% 9.0 9.0 100 53.1 4.2

E&S impact of products and services 17.4% 5.0 5.0 100 43.9 2.8

Human capital 14.4% 4.0 4.0 95 44.4 2.3

Resource use 11.9% 5.0 5.0 80 77.3 1.9

Carbon – own operations 7.6% 6.0 5.7 100 78.6 1.2

Human rights – supply chain 7.3% 4.0 4.2 80 90.0 1.2

Land use and biodiversity – supply 6.0% 3.0 3.5 80 90.0 1.0

chain

Occupational health and safety 5.2% 3.0 3.0 95 75.8 0.8

Resource use – supply chain 2.5% 2.0 2.0 80 100.0 0.4

Business ethics 1.5% 3.0 3.0 95 96.9 0.2

Product governance 0.0% 8.0 8.0 100 100.0 0.0

Overall 100.0% 52.0 52.4 93.5 74.1 16.1

Manageable exposure 49.0


(93.5%)
Managed exposure 36.3
(74.1%)

Within this analysis the:

 Sub-industry exposure indicates the sector average exposure;


 Company exposure indicates the actual exposure of the investee company;
 Manageable risk factor represents the proportion each risk that is capable of being managed by the company;
 Management score represents the proportion of the manageable risk which is managed by the company.

Based on the company's overall exposure of 52.4 and using the other relevant overall exposure figures, what is the ESG risk

rating (the overall unmanaged risk)?

A
The ESG risk rating is 52.4.

B
The ESG risk rating is 49.0.

C
The ESG risk rating is 36.3.
D
The ESG risk rating is 16.1.
You answered : A - The ESG risk rating is 52.4.
The correct answer is: D - The ESG risk rating is 16.1.
Explanation
The company's total exposure is 52.4 and the managed risk is 36.3, hence the ESG risk rating (the overall unmanaged risk) is
16.1 (52.4 – 36.3). The other options are clearly incorrect as they are the total exposure, manageable exposure and managed
exposure contained within the table. See Chapter 7 section 3
Reference: Chapter 7 section 3

Question 25
[4062186]

Several questions are associated with the following case study. The material given in the case study will not change.

You have been asked to prepare a report on the various qualitative or quantitative ways in which ESG factors may be analysed

and integrated into the investment decision making process, culminating in a recommended approach. As a mathematician,

your natural preference is to adopt a quantitative approach, as this easily feeds into your valuation models. However, you are

aware that there are a number of more judgemental qualitative factors which also need to be addressed and which may be

difficult to precisely quantify.

To help with your deliberations you have obtained the following ESG attribution analysis for one of your investee companies

from Sustainalytics and have examined their approach to establishing ESG scores.

Issue Name Contributio Subindustry Company Manageable Management ESG Risk

n to ESG Exposure Exposure Risk Factor Score % Rating

Risk Rating %

Corporate governance 26.2% 9.0 9.0 100 53.1 4.2

E&S impact of products and services 17.4% 5.0 5.0 100 43.9 2.8

Human capital 14.4% 4.0 4.0 95 44.4 2.3

Resource use 11.9% 5.0 5.0 80 77.3 1.9

Carbon – own operations 7.6% 6.0 5.7 100 78.6 1.2

Human rights – supply chain 7.3% 4.0 4.2 80 90.0 1.2

Land use and biodiversity – supply 6.0% 3.0 3.5 80 90.0 1.0

chain

Occupational health and safety 5.2% 3.0 3.0 95 75.8 0.8

Resource use – supply chain 2.5% 2.0 2.0 80 100.0 0.4

Business ethics 1.5% 3.0 3.0 95 96.9 0.2

Product governance 0.0% 8.0 8.0 100 100.0 0.0

Overall 100.0% 52.0 52.4 93.5 74.1 16.1

Manageable exposure 49.0


(93.5%)
Managed exposure 36.3
(74.1%)

Within this analysis the:

 Sub-industry exposure indicates the sector average exposure;


 Company exposure indicates the actual exposure of the investee company;
 Manageable risk factor represents the proportion each risk that is capable of being managed by the company;
 Management score represents the proportion of the manageable risk which is managed by the company.

Based on the company's overall exposure of 52.4 and using the other relevant overall exposure figures, what is the overall

management gap?

A
Management gap is 12.7.

B
Management gap is 36.3.

C
Management gap is 49.0.

D
Management gap is 52.4.
You answered : A - Management gap is 12.7.
The correct answer is: A - Management gap is 12.7.
Explanation
Management gap is the part of the manageable exposure that is not currently managed, i.e 12.7 (49.0 - 36.6). The other options
are clearly incorrect as they are the total exposure, manageable exposure and managed exposure contained within the
table. See Chapter 7 section 3
Reference: Chapter 7 section 5

Question 26
[4062187]

Several questions are associated with the following case study. The material given in the case study will not change.

An institutional investor is considering an investment in an oil company IPO that is being undertaken in an Arabian Gulf-based

developing nation. The country, like many in the region, is largely desert, with the major conurbations on the coast. The oil

extraction facilities are further inland, away from the populated areas.

The country is governed by its ruling family who exercise control through a strong para-military police force and the application

of, on international terms, very strict laws and legal penalties.

The company in question is a state-controlled enterprise which undertakes the management and operational control of all oil

and gas fields within the country starting from the point of extraction and ending at the super tanker that delivers the oil.

The IPO is being undertaken in an effort to reduce the country’s economic exposure to the price of oil and to raise finance that is

to be used to build schools and raise both the standards of education and living standards within the country.

It is the government’s ambition to develop a diversified sovereign wealth fund to help support the economy to the future once oil

production becomes non-viable. With this aim in mind, the government has been investigating the possibility of an IPO for some

time; however it has been delayed over concerns regarding the company’s corporate governance structure and its procurement

policy. These concerns primarily stem from several of the senior board directors having been involved in recent corruption

scandals.
As a reflection of rising geopolitical and military tensions in the region, credit rating agencies have downgraded the country’s

long-term foreign-currency issuer default rating based on their assessment of the vulnerability of the country's economic

infrastructure and the continued deterioration in the country's fiscal and external balance sheets.

To assist with your investment deliberations you have examined various ESG factors for both the company in question and the

country overall using the following scale:

5. Very strong

4. Strong

3. Average

2. Weak

1. Very weak.

Thus, the maximum score is 5 and the minimum score is 1. Based on your assessments, you have constructed the following

scorecard for the company and the country on environmental issues.

Company Country
Emissions 1.6 1.8

Energy efficiency 1.9 2.1

Water scarcity 1.6 1.8

Climate resilience 3.5 3.6

You have also constructed the following scorecard for the company and the country on social issues.

Company Country
Demographics 3.5 3.6

Gender equality 1.9 1.9

Human rights 2.1 2.1

Public safety 2.0 2.0

Education 3.4 3.4

Employment 3.7 3.7

Finally, you have also constructed the following scorecard for the company and the country on governance issues.

Company Country
Political stability 1.9 1.9
Rule of law 1.8 1.8

Corruption levels 1.4 1.6

In respect of greenhouse gas emissions, which of the following would be the most appropriate way in which ESG data should be

disclosed for an oil company if we wish to fully understand the environmental impact of its extraction and use?

A
Scope 1.

B
Scope 2.

C
Scope 3.

D
Scope 4.
You answered : C - Scope 3.
The correct answer is: C - Scope 3.
Explanation
The measurement of greenhouse gas emissions is currently split by the market between Scope 1 (direct emissions from core
operations), Scope 2 (direct emissions from supporting activities, such as from the logistics fleet of a retail chain) and Scope 3
(indirect emissions from the whole value chain, including those produced by suppliers and customers), hence Scope 3 would be
ideal. See Chapter 3 section 2
Reference: Chapter 3 section 2

Question 27
[4062188]

Several questions are associated with the following case study. The material given in the case study will not change.

An institutional investor is considering an investment in an oil company IPO that is being undertaken in an Arabian Gulf-based

developing nation. The country, like many in the region, is largely desert, with the major conurbations on the coast. The oil

extraction facilities are further inland, away from the populated areas.

The country is governed by its ruling family who exercise control through a strong para-military police force and the application

of, on international terms, very strict laws and legal penalties.

The company in question is a state-controlled enterprise which undertakes the management and operational control of all oil

and gas fields within the country starting from the point of extraction and ending at the super tanker that delivers the oil.

The IPO is being undertaken in an effort to reduce the country’s economic exposure to the price of oil and to raise finance that is

to be used to build schools and raise both the standards of education and living standards within the country.

It is the government’s ambition to develop a diversified sovereign wealth fund to help support the economy to the future once oil

production becomes non-viable. With this aim in mind, the government has been investigating the possibility of an IPO for some
time; however it has been delayed over concerns regarding the company’s corporate governance structure and its procurement

policy. These concerns primarily stem from several of the senior board directors having been involved in recent corruption

scandals.

As a reflection of rising geopolitical and military tensions in the region, credit rating agencies have downgraded the country’s

long-term foreign-currency issuer default rating based on their assessment of the vulnerability of the country's economic

infrastructure and the continued deterioration in the country's fiscal and external balance sheets.

To assist with your investment deliberations you have examined various ESG factors for both the company in question and the

country overall using the following scale:

5. Very strong

4. Strong

3. Average

2. Weak

1. Very weak.

Thus, the maximum score is 5 and the minimum score is 1. Based on your assessments, you have constructed the following

scorecard for the company and the country on environmental issues.

Company Country

Emissions 1.6 1.8

Energy efficiency 1.9 2.1

Water scarcity 1.6 1.8

Climate resilience 3.5 3.6

You have also constructed the following scorecard for the company and the country on social issues.
Company Country

Demographics 3.5 3.6

Gender equality 1.9 1.9

Human rights 2.1 2.1

Public safety 2.0 2.0

Education 3.4 3.4

Employment 3.7 3.7

Finally, you have also constructed the following scorecard for the company and the country on governance issues. [

Company Country

Political stability 1.9 1.9

Rule of law 1.8 1.8

Corruption levels 1.4 1.6

An analyst wishes to assess the company’s procurement policies, which of the following should the analyst be looking for in

terms of the policies and systems that the company has in place?

i. Policies which clearly explain the environmental and social requirements that suppliers are expected to meet via a

procurement policy, such as a supplier code of conduct.

ii. Systems which enable it to assess environmental and social risks throughout its supply chain.

iii. A mechanism in place to improve poor practices

A
i. and ii. only.

B
i. and iii. only.

C
ii. and iii. only.

D
i., ii. and iii.
You answered : D - i., ii. and iii.
The correct answer is: D - i., ii. and iii.
Explanation
Procurement policies and systems should clearly explain the environmental and social requirements that suppliers are expected
to meet via a procurement policy, such as a supplier code of conduct. Procurement systems should also allow it to assess
environmental and social risks throughout its supply chain, and a mechanism should exist to improve poor practices. See
Chapter 3 Section 2
Reference: Chapter 3 section 2

Question 28
[4062189]

Several questions are associated with the following case study. The material given in the case study will not change.

An institutional investor is considering an investment in an oil company IPO that is being undertaken in an Arabian Gulf-based

developing nation. The country, like many in the region, is largely desert, with the major conurbations on the coast. The oil

extraction facilities are further inland, away from the populated areas.

The country is governed by its ruling family who exercise control through a strong para-military police force and the application

of, on international terms, very strict laws and legal penalties.

The company in question is a state-controlled enterprise which undertakes the management and operational control of all oil

and gas fields within the country starting from the point of extraction and ending at the super tanker that delivers the oil.

The IPO is being undertaken in an effort to reduce the country’s economic exposure to the price of oil and to raise finance that is

to be used to build schools and raise both the standards of education and living standards within the country.

It is the government’s ambition to develop a diversified sovereign wealth fund to help support the economy to the future once oil

production becomes non-viable. With this aim in mind, the government has been investigating the possibility of an IPO for some

time; however it has been delayed over concerns regarding the company’s corporate governance structure and its procurement

policy. These concerns primarily stem from several of the senior board directors having been involved in recent corruption

scandals.

As a reflection of rising geopolitical and military tensions in the region, credit rating agencies have downgraded the country’s

long-term foreign-currency issuer default rating based on their assessment of the vulnerability of the country's economic

infrastructure and the continued deterioration in the country's fiscal and external balance sheets.

To assist with your investment deliberations you have examined various ESG factors for both the company in question and the

country overall using the following scale:

5. Very strong

4. Strong

3. Average

2. Weak

1. Very weak.
Thus, the maximum score is 5 and the minimum score is 1. Based on your assessments, you have constructed

the following scorecard for the company and the country on environmental issues.

Company Country
Emissions 1.6 1.8

Energy efficiency 1.9 2.1

Water scarcity 1.6 1.8

Climate resilience 3.5 3.6

You have also constructed the following scorecard for the company and the country on social issues.

Company Country
Demographics 3.5 3.6

Gender equality 1.9 1.9

Human rights 2.1 2.1

Public safety 2.0 2.0

Education 3.4 3.4

Employment 3.7 3.7

Finally, you have also constructed the following scorecard for the company and the country on governance issues.

Company Country
Political stability 1.9 1.9

Rule of law 1.8 1.8

Corruption levels 1.4 1.6

How would you best characterise the country in question in terms of environmental issues and factors?
A
The country has poor energy efficiency and is a high emitter of greenhouse gasses. It suffers from water scarcity and has poor
climate resilience.

B
The country has good energy efficiency and is a low emitter of greenhouse gasses. It suffers from water scarcity but has
reasonable climate resilience.

C
The country has poor energy efficiency and is a high emitter of greenhouse gasses. It suffers from water scarcity but has
reasonable climate resilience.
D
The country has good energy efficiency and is a low emitter of greenhouse gasses. It suffers from water scarcity and has poor
climate resilience.
You answered : C - The country has poor energy efficiency and is a high emitter of greenhouse gasses. It suffers from water
scarcity but has reasonable climate resilience.
The correct answer is: C - The country has poor energy efficiency and is a high emitter of greenhouse gasses. It suffers from
water scarcity but has reasonable climate resilience.
Explanation
Based on the environmental assessment table and the scoring system used, the country has poor energy efficiency and is a
high emitter of greenhouse gasses. It suffers from water scarcity but has reasonable climate resilience. See Chapter 3 Section 6
Reference: Chapter 3 section 6

Question 29
[4062190]

Several questions are associated with the following case study. The material given in the case study will not change.

An institutional investor is considering an investment in an oil company IPO that is being undertaken in an Arabian Gulf-based
developing nation. The country, like many in the region, is largely desert, with the major conurbations on the

coast. The oil extraction facilities are further inland, away from the populated areas.

The country is governed by its ruling family who exercise control through a strong para-military police force and

the application of, on international terms, very strict laws and legal penalties.

The company in question is a state-controlled enterprise which undertakes the management and operational

control of all oil and gas fields within the country starting from the point of extraction and ending at the super

tanker that delivers the oil.

The IPO is being undertaken in an effort to reduce the country’s economic exposure to the price of oil and to

raise finance that is to be used to build schools and raise both the standards of education and living standards

within the country.

It is the government’s ambition to develop a diversified sovereign wealth fund to help support the economy to

the future once oil production becomes non-viable. With this aim in mind, the government has been

investigating the possibility of an IPO for some time; however it has been delayed over concerns regarding the

company’s corporate governance structure and its procurement policy. These concerns primarily stem from

several of the senior board directors having been involved in recent corruption scandals.

As a reflection of rising geopolitical and military tensions in the region, credit rating agencies have downgraded

the country’s long-term foreign-currency issuer default rating based on their assessment of the vulnerability of

the country's economic infrastructure and the continued deterioration in the country's fiscal and external

balance sheets. T

o assist with your investment deliberations you have examined various ESG factors for both the company in

question and the country overall using the following scale:

5. Very strong

4. Strong

3. Average
2. Weak

1. Very weak

Thus, the maximum score is 5 and the minimum score is 1. Based on your assessments, you have constructed

the following scorecard for the company and the country on environmental issues.

Company Country
Emissions 1.6 1.8

Energy efficiency 1.9 2.1

Water scarcity 1.6 1.8

Climate resilience 3.5 3.6

You have also constructed the following scorecard for the company and the country on social issues.

Company Country
Demographics 3.5 3.6

Gender equality 1.9 1.9

Human rights 2.1 2.1

Public safety 2.0 2.0

Education 3.4 3.4

Employment 3.7 3.7

Finally, you have also constructed the following scorecard for the company and the country on governance

issues.

Company Country
Political stability 1.9 1.9

Rule of law 1.8 1.8

Corruption levels 1.4 1.6

How would you best characterise the company in question in terms of social and governance factors, and which

factor would you consider to be the greatest investment concern?


A
The company has a reasonable record on human rights, public safety, gender equality and governance, with gender equality
being the greatest investment concern.

B
The company has a reasonable record on human rights, public safety, but a poor record on gender equality and governance,
with corruption being the greatest investment concern.

C
The company has a poor record on human rights, public safety, gender equality and governance, with the potential for
corruption being the greatest investment concern.

D
The company has a reasonable record on human rights, public safety, but a poor record on gender equality and governance,
with gender equality being the greatest investment concern.
You answered : C - The company has a poor record on human rights, public safety, gender equality and governance, with
the potential for corruption being the greatest investment concern.
The correct answer is: C - The company has a poor record on human rights, public safety, gender equality and governance,
with the potential for corruption being the greatest investment concern.
Explanation
Based on the social and governance assessment tables and the scoring system used, the company has a poor record on
human rights, public safety, gender equality and governance, with the potential for corruption being the greatest investment
concern (having the lowest score). See Chapter 7 section 3
Reference: Chapter 7 section 3

Question 30
[4062191]

Several questions are associated with the following case study. The material given in the case study will not change.

An institutional investor is considering an investment in an oil company IPO that is being undertaken in an Arabian Gulf-based

developing nation. The country, like many in the region, is largely desert, with the major conurbations on the coast. The oil

extraction facilities are further inland, away from the populated areas.

The country is governed by its ruling family who exercise control through a strong para-military police force and the application

of, on international terms, very strict laws and legal penalties. The company in question is a state-controlled enterprise which

undertakes the management and operational control of all oil and gas fields within the country starting from the point of

extraction and ending at the super tanker that delivers the oil.

The IPO is being undertaken in an effort to reduce the country’s economic exposure to the price of oil and to raise finance that is

to be used to build schools and raise both the standards of education and living standards within the country.

It is the government’s ambition to develop a diversified sovereign wealth fund to help support the economy to the future once oil

production becomes non-viable. With this aim in mind, the government has been investigating the possibility of an IPO for some

time; however it has been delayed over concerns regarding the company’s corporate governance structure and its procurement

policy. These concerns primarily stem from several of the senior board directors having been involved in recent corruption

scandals.
As a reflection of rising geopolitical and military tensions in the region, credit rating agencies have downgraded the country’s

long-term foreign-currency issuer default rating based on their assessment of the vulnerability of the country's economic

infrastructure and the continued deterioration in the country's fiscal and external balance sheets.

To assist with your investment deliberations you have examined various ESG factors for both the company in question and the

country overall using the following scale:

5. Very strong

4. Strong

3. Average

2. Weak

1. Very weak.

Thus, the maximum score is 5 and the minimum score is 1. Based on your assessments, you have constructed the following

scorecard for the company and the country on environmental issues.

Company Country

Emissions 1.6 1.8

Energy efficiency 1.9 2.1

Water scarcity 1.6 1.8

Climate resilience 3.5 3.6

You have also constructed the following scorecard for the company and the country on social issues.

Company Country

Demographics 3.5 3.6

Gender equality 1.9 1.9


Human rights 2.1 2.1

Public safety 2.0 2.0

Education 3.4 3.4

Employment 3.7 3.7

Finally, you have also constructed the following scorecard for the company and the country on governance issues.

Company Country

Political stability 1.9 1.9

Rule of law 1.8 1.8

Corruption levels 1.4 1.6

One of the directors of the company has stated that, as the original extractors of the oil and gas, they have no supply chain and

are based in an arid desert region. As a result, they have no direct or indirect impact on biodiversity. How would you view their

impact on diversity?

A
They have neither a direct nor an indirect impact.

B
They don’t have a direct impact but do have an indirect impact.

C
They have a direct impact but don’t have an indirect impact.

D
They have both a direct and an indirect impact.
You answered : B - They don’t have a direct impact but do have an indirect impact.
The correct answer is: D - They have both a direct and an indirect impact.
Explanation
A direct impact is the way in which an organisation’s activities directly affecting biodiversity. Although the oil wells may be in a
desert, the company's operations run to the super tanker at sea where biodiversity will be significantly affected. An indirect
impact is the impact caused by parties in an organisation’s supply chain. Oil companies have significant supply chains for their
infrastructure components which, again, run to the sea. As such, they are liable to have a significant indirect impact. See
Chapter 3 aection 2
Reference: Chapter 3 section 3

Question 31
[4062192]

Several questions are associated with the following case study. The material given in the case study will not change.
A white goods manufacturing business based in Germany wishes to incorporate new technology into its products that will

enable them to monitor the energy efficiency of their devices and remotely identify maintenance issues at an early stage before

they actually manifest themselves to the consumer. The company is currently investing heavily in automated manufacturing and

monitoring infrastructure to streamline their operations.

To achieve this, the device must be fitted with wireless network technology which captures data on the device and how it is

running. The manufacturer is aware that such technology already exists and is available off-the-shelf from a technology provider

for incorporation into their product.

Concerns have, however, been raised regarding the potential for the device to capture additional information without the

consumer’s consent, raising potentially data protection concerns for the company.

The concerns primarily stem from the technology provider being based in China, having been founded in 1987 by an electronic

engineer who was formerly an engineer in the People's Liberation Army. His connections to the military and to the Communist

Party, alongside those of other senior executives, have been cited as a security concern for foreign customers.

The technology provider has confirmed that the company should have no data protection concerns but have refused to provide

access to provide access to the systems or software in order to allow confirmation, citing intellectual property concerns.

The manufacturer’s internal audit team have also expressed particular concerns over the quality of the technology provider’s

governance. Concerns have also been expressed with respect to human rights, employment rights, corruption and the absence

of competition within China.

To counter these suggestions, the technology provider has tried to promote its openness as evidenced by the levels of

disclosure in its financial statements and the cleanliness of its audit report.

The manufacturer has no control over the functioning of the off-the-shelf wireless network technology but believe that they have

found a software solution to the data security concerns. Consequently, despite the noted concerns the manufacturer is keen to

press ahead with their technologically advanced product.

Which of the following BEST describes the role of the auditor in relation to the financial statements and to the narrative elements

of the annual report and accounts?

A
The auditor provides assurance on the financial statements but has no responsibility with respect to the narrative reports.
B
The auditor provides assurance on the financial statements and reports any inconsistencies in the narrative reports.

C
The auditor provides assurance on the financial statements and on any financial figures contained in the narrative reports.

D
The auditor provides assurance on both the financial statements and the narrative reports.
You answered : C - The auditor provides assurance on the financial statements and on any financial figures contained in the
narrative reports.
The correct answer is: B - The auditor provides assurance on the financial statements and reports any inconsistencies in the
narrative reports.
Explanation
The auditor is there to provide an independent pair of eyes assessing the financial reports prepared by management, and to
provide some assurance that those reports fairly represent the performance and position of the business. There is no absolute
assurance that the numbers are correct, nor is there certainty that there is no fraud within the business. The auditor checks and
assures the financial statements in detail; their role in relation to the words and numbers in the more discursive front half of the
annual report is less stringent. The auditor must read this segment and should comment if they discover something that is
inconsistent with what they have learned through the process of the audit. See Chapter 5 Section 6
Reference: Chapter 5 section 6

Question 32
[4062193]

Several questions are associated with the following case study. The material given in the case study will not change.

A white goods manufacturing business based in Germany wishes to incorporate new technology into its products that will

enable them to monitor the energy efficiency of their devices and remotely identify maintenance issues at an early stage before

they actually manifest themselves to the consumer. The company is currently investing heavily in automated manufacturing and

monitoring infrastructure to streamline their operations.

To achieve this, the device must be fitted with wireless network technology which captures data on the device and how it is

running. The manufacturer is aware that such technology already exists and is available off-the-shelf from a technology provider

for incorporation into their product.

Concerns have, however, been raised regarding the potential for the device to capture additional information without the

consumer’s consent, raising potentially data protection concerns for the company.

The concerns primarily stem from the technology provider being based in China, having been founded in 1987 by an electronic

engineer who was formerly an engineer in the People's Liberation Army. His connections to the military and to the Communist

Party, alongside those of other senior executives, have been cited as a security concern for foreign customers.

The technology provider has confirmed that the company should have no data protection concerns but have refused to provide

access to provide access to the systems or software in order to allow confirmation, citing intellectual property concerns.

The manufacturer’s internal audit team have also expressed particular concerns over the quality of the technology provider’s

governance. Concerns have also been expressed with respect to human rights, employment rights, corruption and the absence

of competition within China.

To counter these suggestions, the technology provider has tried to promote its openness as evidenced by the levels of

disclosure in its financial statements and the cleanliness of its audit report.

The manufacturer has no control over the functioning of the off-the-shelf wireless network technology but believe that they have

found a software solution to the data security concerns. Consequently, despite the noted concerns the manufacturer is keen to

press ahead with their technologically advanced product.


From the perspective of the German manufacturer, which of the following would most likely describe the rational assessment of

the governance risk associated with the technology provider?

A
Governance risk is low as the technology provider is open in its accounting disclosures and it has a clean audit report.

B
Governance risk is a minor concern as the component is just one smal element of the product.

C
Governance risk is moderate but controllable through the use of the German company's technology.

D
Governance risk is high as the manufacturer has no control over the data that may be captured nor how it may be used.
You answered : D - Governance risk is high as the manufacturer has no control over the data that may be captured nor how
it may be used.
The correct answer is: D - Governance risk is high as the manufacturer has no control over the data that may be captured
nor how it may be used.
Explanation
Governance risk is high as the manufacturer has no control over the data that may be captured nor how it may be used. There
is clear potential for corporate abuse. See Chapter 5 section 7
Reference: Chapter 5 section 7

Question 33
[4062194]

Several questions are associated with the following case study. The material given in the case study will not change.

A white goods manufacturing business based in Germany wishes to incorporate new technology into its products that will

enable them to monitor the energy efficiency of their devices and remotely identify maintenance issues at an early stage before

they actually manifest themselves to the consumer. The company is currently investing heavily in automated manufacturing and

monitoring infrastructure to streamline their operations.

To achieve this, the device must be fitted with wireless network technology which captures data on the device and how it is

running. The manufacturer is aware that such technology already exists and is available off-the-shelf from a technology provider

for incorporation into their product.

Concerns have, however, been raised regarding the potential for the device to capture additional information without the

consumer’s consent, raising potentially data protection concerns for the company.

The concerns primarily stem from the technology provider being based in China, having been founded in 1987 by an electronic

engineer who was formerly an engineer in the People's Liberation Army. His connections to the military and to the Communist

Party, alongside those of other senior executives, have been cited as a security concern for foreign customers.
The technology provider has confirmed that the company should have no data protection concerns but have refused to provide

access to provide access to the systems or software in order to allow confirmation, citing intellectual property concerns.

The manufacturer’s internal audit team have also expressed particular concerns over the quality of the technology provider’s

governance. Concerns have also been expressed with respect to human rights, employment rights, corruption and the absence

of competition within China.

To counter these suggestions, the technology provider has tried to promote its openness as evidenced by the levels of

disclosure in its financial statements and the cleanliness of its audit report. The manufacturer has no control over the functioning

of the off-the-shelf wireless network technology but believe that they have found a software solution to the data security

concerns. Consequently, despite the noted concerns the manufacturer is keen to press ahead with their technologically

advanced product.

Given their countries of incorporation, what is the most likely formal board structure of both the manufacturer and the technology

provider?

A
Both will have a unitary board.

B
The manufacturer will have a unitary board, the technology provider will have a two-tier board.

C
The manufacturer will have a two-tier board, the technology provider will have a unitary board.

D
Both will have a two-tier board.
You answered : B - The manufacturer will have a unitary board, the technology provider will have a two-tier board.
The correct answer is: D - Both will have a two-tier board.
Explanation
The division between the supervisory board and the management board marks one of the fundamental structural differences in
governance globally, between these so-called two-tier board structures seen, for example, in Germany, the Netherlands,
Scandinavia and (at least formally) China, and the single-tier (also called unitary) boards that are more typical of the UK, the
USA, Japan, France and most of the rest of the world. See Chapter 5 section 5
Reference: Chapter 5 section 5

Question 34
[4062195]

Several questions are associated with the following case study. The material given in the case study will not change.

A white goods manufacturing business based in Germany wishes to incorporate new technology into its products that will

enable them to monitor the energy efficiency of their devices and remotely identify maintenance issues at an early stage before

they actually manifest themselves to the consumer. The company is currently investing heavily in automated manufacturing and

monitoring infrastructure to streamline their operations.


To achieve this, the device must be fitted with wireless network technology which captures data on the device and how it is

running. The manufacturer is aware that such technology already exists and is available off-the-shelf from a technology provider

for incorporation into their product.

Concerns have, however, been raised regarding the potential for the device to capture additional information without the

consumer’s consent, raising potentially data protection concerns for the company.

The concerns primarily stem from the technology provider being based in China, having been founded in 1987 by an electronic

engineer who was formerly an engineer in the People's Liberation Army. His connections to the military and to the Communist

Party, alongside those of other senior executives, have been cited as a security concern for foreign customers.

The technology provider has confirmed that the company should have no data protection concerns but have refused to provide

access to provide access to the systems or software in order to allow confirmation, citing intellectual property concerns.

The manufacturer’s internal audit team have also expressed particular concerns over the quality of the technology provider’s

governance. Concerns have also been expressed with respect to human rights, employment rights, corruption and the absence

of competition within China.

To counter these suggestions, the technology provider has tried to promote its openness as evidenced by the levels of

disclosure in its financial statements and the cleanliness of its audit report.

The manufacturer has no control over the functioning of the off-the-shelf wireless network technology but believe that they have

found a software solution to the data security concerns. Consequently, despite the noted concerns the manufacturer is keen to

press ahead with their technologically advanced product.

If they go ahead with the product, what ESG risks, if any, will the manufacturer face in respect of data

protection?
A
The manufacturer will not face any risks as they believe they have a suitable software solution.

B
The manufacturer will face an environmental risk.

C
The manufacturer will face a social risk.

D
The manufacturer will face an environmental risk.
You answered : C - The manufacturer will face a social risk.
The correct answer is: C - The manufacturer will face a social risk.
Explanation
As they have no control over the functioning of the off-the-shelf wireless network technology and only believe that they have
found a software solution to the data security concerns, the manufacturer is exposed to a potential data security risk. This is
classified as a social risk. See Chapter 7 section 3
Reference: Chapter 7 section 3

Question 35
[4062196]

Several questions are associated with the following case study. The material given in the case study will not change.

A white goods manufacturing business based in Germany wishes to incorporate new technology into its products that will

enable them to monitor the energy efficiency of their devices and remotely identify maintenance issues at an early stage before

they actually manifest themselves to the consumer. The company is currently investing heavily in automated manufacturing and

monitoring infrastructure to streamline their operations.

To achieve this, the device must be fitted with wireless network technology which captures data on the device and how it is

running. The manufacturer is aware that such technology already exists and is available off-the-shelf from a technology provider

for incorporation into their product.

Concerns have, however, been raised regarding the potential for the device to capture additional information without the

consumer’s consent, raising potentially data protection concerns for the company.

The concerns primarily stem from the technology provider being based in China, having been founded in 1987 by an electronic

engineer who was formerly an engineer in the People's Liberation Army. His connections to the military and to the Communist

Party, alongside those of other senior executives, have been cited as a security concern for foreign customers.

The technology provider has confirmed that the company should have no data protection concerns but have refused to provide

access to provide access to the systems or software in order to allow confirmation, citing intellectual property concerns.

The manufacturer’s internal audit team have also expressed particular concerns over the quality of the technology provider’s

governance. Concerns have also been expressed with respect to human rights, employment rights, corruption and the absence

of competition within China.

To counter these suggestions, the technology provider has tried to promote its openness as evidenced by the levels of

disclosure in its financial statements and the cleanliness of its audit report.
The manufacturer has no control over the functioning of the off-the-shelf wireless network technology but believe that they have

found a software solution to the data security concerns. Consequently, despite the noted concerns the manufacturer is keen to

press ahead with their technologically advanced product.

If the software solution to the security concerns is ineffective but the manufacturer still presses ahead with production, how

would a purely qualitative equity investor incorporate ESG risk factors into their investment decision framework?

A
Qualitative adjustment to relative valuation models

B
Qualitative adjustment to forecast cash flows

C
Qualitative adjustment to the cost of capital

D
Qualitative adjustment at the end of the analysis process
You answered : C - Qualitative adjustment to the cost of capital
The correct answer is: D - Qualitative adjustment at the end of the analysis process
Explanation
ESG factors may be integrated in either: a purely qualitative manner, e.g. opinion on quality of management added to the
investment thesis (the reasoned argument for a particular investment strategy); or in a quantitative manner, e.g. impact on
forecast profits, valuation models/valuation ratios, etc. Some techniques could be considered a hybrid of both techniques, for
example, scorecards, where a qualitative judgment is turned into a quantitative score. See Chapter 7 section 2
Reference: Chapter 7 section 2

Question 36
[4062197]

Several questions are associated with the following case study. The material given in the case study will not change.

A US municipal authority based in a city in Pennsylvania wishes to undertake a municipal green bond issue to finance its
highway and transit system construction projects. The municipal authority is an independent corporate agent of the

Commonwealth of Pennsylvania, exercising governmental, as well as private corporate power, in assisting the

Commonwealth in meeting the needs of its citizens. The authority in question has a history of bribery and

corruption, although the current executives have not been in-situ for long and suggest this to be a historic

anomaly that is no longer relevant to the authority's operations.

The authority's plan is to update the existing road network with a more automated and coordinated traffic

control system that makes use of AI to predict and ameliorate the impact of any congestion as and when it

begins to arise. The technology is still in development; however the results to date are very positive.

The authority also intends to implement a public transit system, the current public transport system being

reliant on diesel-powered busses. This system is to be entirely electrified, powered from a renewable energy

source, and able to efficiently transport citizens from the outer urban areas into the commercial district each

day.
The integrates transit system will be based around a driverless subway system with the existing bus network

being reconfigured to transport citizens to and from local transit stations, thereby reducing city centre

congestion levels. The existing busses will also be updated to plug-in hybrid technology. Construction will make

extensive use of an existing disused rail system consisting of a number of tunnels underneath the city, although

this will need extensive refurbishment.

Construction will, however, rely on nearby steel manufacturer which is known to pollute the local water basin.

There will also be a heavy demand for concrete, which is considered to be one of the most environmentally

destructive materials, being responsible for 4%-8% of global CO2 emissions.

As a credit analyst examining the ESG aspects of the potential bond issue, what would you consider to be the

lowest ESG risk in relation to the transit system give that it is to be powered by renewable energy?
A
Environmental factors.

B
Social factors.

C
Governance factors.

D
There will be no ESG risks.
You answered : D - There will be no ESG risks.
The correct answer is: B - Social factors.
Explanation
The purpose of the bond issue is to finance construction; hence we need to consider the ESG issues that may arise from this
activity. There is a clear governance risk given the authority’s poor record on bribery and corruption. There is also a clear
environmental risk that will arise from the construction work. In contrast, material social risks appear to be relatively limited. See
Chapter 7 section 3
Reference: Chapter 7 section 3

Question 37
[4062198]

Several questions are associated with the following case study. The material given in the case study will not change.

A US municipal authority based in a city in Pennsylvania wishes to undertake a municipal green bond issue to finance its

highway and transit system construction projects. The municipal authority is an independent corporate agent of the

Commonwealth of Pennsylvania, exercising governmental, as well as private corporate power, in assisting the Commonwealth

in meeting the needs of its citizens. The authority in question has a history of bribery and corruption, although the current

executives have not been in-situ for long and suggest this to be a historic anomaly that is no longer relevant to the authority's

operations.

The authority's plan is to update the existing road network with a more automated and coordinated traffic control system that

makes use of AI to predict and ameliorate the impact of any congestion as and when it begins to arise. The technology is still in

development; however the results to date are very positive.


The authority also intends to implement a public transit system, the current public transport system being reliant on diesel-

powered busses. This system is to be entirely electrified, powered from a renewable energy source, and able to efficiently

transport citizens from the outer urban areas into the commercial district each day.

The integrates transit system will be based around a driverless subway system with the existing bus network being reconfigured

to transport citizens to and from local transit stations, thereby reducing city centre congestion levels. The existing busses will

also be updated to plug-in hybrid technology. Construction will make extensive use of an existing disused rail system consisting

of a number of tunnels underneath the city, although this will need extensive refurbishment.

Construction will, however, rely on nearby steel manufacturer which is known to pollute the local water basin. There will also be

a heavy demand for concrete, which is considered to be one of the most environmentally destructive materials, being

responsible for 4%-8% of global CO2 emissions.

Based on the CICERO shades of green methodology that determine how a bond aligns with a low-carbon resilient future, how

would the transit system be classified?

A
Dark green.

B
Blue.

C
Brown.

D
Light green.
You answered : D - Light green.
The correct answer is: A - Dark green.
Explanation
With respect to green bonds, dark green is allocated to projects and solutions that correspond to the long-term vision of a low
carbon and climate resilient future, which the transit system represents. See Chapter 3 section 7
Reference: Chapter 3 section 7

Question 38
[4062199]

Several questions are associated with the following case study. The material given in the case study will not change.

A US municipal authority based in a city in Pennsylvania wishes to undertake a municipal green bond issue to finance its

highway and transit system construction projects. The municipal authority is an independent corporate agent of the

Commonwealth of Pennsylvania, exercising governmental, as well as private corporate power, in assisting the Commonwealth

in meeting the needs of its citizens. The authority in question has a history of bribery and corruption, although the current

executives have not been in-situ for long and suggest this to be a historic anomaly that is no longer relevant to the authority's

operations.
The authority's plan is to update the existing road network with a more automated and coordinated traffic control system that

makes use of AI to predict and ameliorate the impact of any congestion as and when it begins to arise. The technology is still in

development; however the results to date are very positive.

The authority also intends to implement a public transit system, the current public transport system being reliant on diesel-

powered busses. This system is to be entirely electrified, powered from a renewable energy source, and able to efficiently

transport citizens from the outer urban areas into the commercial district each day.

The integrates transit system will be based around a driverless subway system with the existing bus network being reconfigured

to transport citizens to and from local transit stations, thereby reducing city centre congestion levels. The existing busses will

also be updated to plug-in hybrid technology. Construction will make extensive use of an existing disused rail system consisting

of a number of tunnels underneath the city, although this will need extensive refurbishment.

Construction will, however, rely on nearby steel manufacturer which is known to pollute the local water basin. There will also be

a heavy demand for concrete, which is considered to be one of the most environmentally destructive materials, being

responsible for 4%-8% of global CO2 emissions.

Based on the CICERO shades of green methodology that determine how a bond aligns with a low-carbon resilient future, how

would the reconfigured bus system be classified?

A
Dark green.

B
Medium green.

C
White.

D
Light green
You answered : B - Medium green.
The correct answer is: B - Medium green.
Explanation
Medium green covers bridging technology such as plug-in hybrid busses. Medium green is allocated to projects and solutions
that represent steps towards the long-term vision but are not quite there yet. See Chapter 3 section 7
Reference: Chapter 3 section 7

Question 39
[4062200]

Several questions are associated with the following case study. The material given in the case study will not change.
A US municipal authority based in a city in Pennsylvania wishes to undertake a municipal green bond issue to finance its

highway and transit system construction projects. The municipal authority is an independent corporate agent of the

Commonwealth of Pennsylvania, exercising governmental, as well as private corporate power, in assisting the Commonwealth

in meeting the needs of its citizens. The authority in question has a history of bribery and corruption, although the current

executives have not been in-situ for long and suggest this to be a historic anomaly that is no longer relevant to the authority's

operations.

The authority's plan is to update the existing road network with a more automated and coordinated traffic control system that

makes use of AI to predict and ameliorate the impact of any congestion as and when it begins to arise. The technology is still in

development; however the results to date are very positive.

The authority also intends to implement a public transit system, the current public transport system being reliant on diesel-

powered busses. This system is to be entirely electrified, powered from a renewable energy source, and able to efficiently

transport citizens from the outer urban areas into the commercial district each day.

The integrates transit system will be based around a driverless subway system with the existing bus network being reconfigured

to transport citizens to and from local transit stations, thereby reducing city centre congestion levels. The existing busses will

also be updated to plug-in hybrid technology. Construction will make extensive use of an existing disused rail system consisting

of a number of tunnels underneath the city, although this will need extensive refurbishment.

Construction will, however, rely on nearby steel manufacturer which is known to pollute the local water basin.
There will also be a heavy demand for concrete, which is considered to be one of the most environmentally

destructive materials, being responsible for 4%-8% of global CO2 emissions.

Which body established the Green Bond Principles (GBP)?

A
The United Nations Environment Programme (UNEP).

B
The International Capital Markets Association (ICMA).

C
The Stockholm Resilience Centre (SRC).

D
The Intergovernmental Panel on Climate Change (IPCC).
You answered : B - The International Capital Markets Association (ICMA).
The correct answer is: B - The International Capital Markets Association (ICMA).
Explanation
The International Capital Markets Association (ICMA) sets out voluntary best practice guidelines called the Green Bond
Principles (GBP), which were established in 2014 by a consortium of investment banks to promote the integrity of the green
bond market by recommending transparency, disclosure and reporting. See Chapter 3 section 7
Reference: Chapter 3 section 7

Question 40
[4062201]

Several questions are associated with the following case study. The material given in the case study will not change.

A US municipal authority based in a city in Pennsylvania wishes to undertake a municipal green bond issue to finance its

highway and transit system construction projects. The municipal authority is an independent corporate agent of the

Commonwealth of Pennsylvania, exercising governmental, as well as private corporate power, in assisting the Commonwealth

in meeting the needs of its citizens. The authority in question has a history of bribery and corruption, although the current

executives have not been in-situ for long and suggest this to be a historic anomaly that is no longer relevant to the authority's

operations.

The authority's plan is to update the existing road network with a more automated and coordinated traffic control system that

makes use of AI to predict and ameliorate the impact of any congestion as and when it begins to arise. The technology is still in

development; however the results to date are very positive.

The authority also intends to implement a public transit system, the current public transport system being reliant on diesel-

powered busses. This system is to be entirely electrified, powered from a renewable energy source, and able to efficiently

transport citizens from the outer urban areas into the commercial district each day.

The integrates transit system will be based around a driverless subway system with the existing bus network being reconfigured

to transport citizens to and from local transit stations, thereby reducing city centre congestion levels. The existing busses will

also be updated to plug-in hybrid technology. Construction will make extensive use of an existing disused rail system consisting

of a number of tunnels underneath the city, although this will need extensive refurbishment.

Construction will, however, rely on nearby steel manufacturer which is known to pollute the local water basin. There will also be

a heavy demand for concrete, which is considered to be one of the most environmentally destructive materials, being

responsible for 4%-8% of global CO2 emissions.

Which of the following is NOT one of the four key pillars of the Green Loan Principles (GLP)?

A
There is clear green use of loan proceeds.

B
The project's sustainability objectives have been clearly evaluated and communicated to lenders.
C
Loan proceeds are strictly managed through project accounts.

D
Reporting is mandated and detailed but offers some flexibility.
You answered : D - Reporting is mandated and detailed but offers some flexibility.
The correct answer is: D - Reporting is mandated and detailed but offers some flexibility.
Explanation
The four key pillars of the Green Loan Principles (GLP) are: there is clear green use of loan proceeds; the project's sustainability
objectives have been clearly evaluated and communicated to lenders; loan proceeds are strictly managed through project
accounts; reporting is mandated, detailed and strict. See Chapter 3 section 7
Reference: Chapter 3 section 7

Question 41
[4030405] Which of the following asset classes has seen the lowest investment manager progress on ESG integration?
A
Public equities.

B
Infrastructure.

C
Fixed income.

D
Hedge funds.
You answered : D - Hedge funds.
The correct answer is: D - Hedge funds.
Explanation
Hedge funds have seen the lowest manager progress on ESG integration.
Reference: Chapter 8 section 1

Question 42
[4030406] Which of the following most accurately describes the most commonly taken approach to discretionary ESG
investment strategies?
A
A fundamental portfolio approach to complement bottom-up financial analysis, alongside the consideration of ESG factors.

B
A quantitative portfolio approach to complement bottom-up financial analysis, alongside the consideration of ESG factors.

C
A fundamental portfolio approach to complement top-down financial analysis, alongside the consideration of ESG factors.

D
A quantitative portfolio approach to complement top-down financial analysis, alongside the consideration of ESG factors.
You answered : A - A fundamental portfolio approach to complement bottom-up financial analysis, alongside the
consideration of ESG factors.
The correct answer is: A - A fundamental portfolio approach to complement bottom-up financial analysis, alongside the
consideration of ESG factors.
Explanation
Discretionary ESG investment strategies most commonly take the form of a fundamental portfolio approach. A portfolio manager
would work to complement bottom-up financial analysis alongside the consideration of ESG factors to reinforce the investment
thesis of a particular holding.
Reference: Chapter 8 section 1

Question 43
[4030407] Which of the following rates more than 4,000 securities in developed and emerging securities on 300 ESG
indicators?
A
FTSE Russell.

B
FTSE4Good.
C
JP Morgan ESG EMD.

D
MSCI ESG
You answered : D - MSCI ESG
The correct answer is: A - FTSE Russell.
Explanation
The FTSE Russell indices rates above 4,000 securities in developed and emerging countries on 300 ESG indicators, and
measures companies' revenue exposure and management to green and brown (fossil fuel) exposure.
Reference: Chapter 8 section 2

Question 44
[4030408] For asset owners and multi-asset managers, which of the following may account for as much as 90% of the variability
in investment returns of a typical fund over time.
A
Stock selection.

B
Tactical asset allocation.

C
Strategic asset allocation.

D
Immunisation.
You answered : A - Stock selection.
The correct answer is: C - Strategic asset allocation.
Explanation
For asset owners and multi-asset managers, strategic asset allocation represents the most important, top-down decision that
will carry wide-ranging implications depending on exposure to asset classes and investment strategy types. Indeed, the
strategic asset allocation policy may account for as much as 90% of the variability in investment returns of a typical fund over
time.
Reference: Chapter 8 section 3

Question 45
[4030409] Which of the following strategic asset allocation models is driven by changes in risk tolerance, typically induced by
cumulative performance relative to investment goals or an approaching investment horizon?
A
Liability driven asset allocation.

B
Dynamic asset allocation (DAA).

C
Factor risk allocation.

D
Mean-variance optimisation (MVO).
You answered : B - Dynamic asset allocation (DAA).
The correct answer is: B - Dynamic asset allocation (DAA).
Explanation
Dynamic asset allocation (DAA) is driven by changes in risk tolerance, typically induced by cumulative performance relative to
investment goals or an approaching investment horizon.
Reference: Chapter 8 section 3

Question 46
[4030410] Which of the following has emerged as the most material ESG factor for institutional investors to address within asset
allocation strategies?
A
Climate change.

B
Pollution.

C
Human rights.

D
Board diversity.
You answered : B - Pollution.
The correct answer is: A - Climate change.
Explanation
Climate change and climate risk has emerged as the most material ESG factor for institutional investors to address within asset
allocation strategies. Climate risk is both systemic and local. It threatens the financial system and the global means of
production as much as it poses risk on a more localised level for specific regions, sectors and companies. Its potential physical
risks will manifest in both acute, event-driven forms (such as extreme weather) and longer-term, chronic shifts driven by the
effects of elevated temperatures and rising sea levels.
Reference: Chapter 8 section 3

Question 47
[4030411] LGT Capital Partners has tracked the development of ESG capabilities among its managers for seven years. What
has the data illustrated?
A
A deteriorating trend in ESG capabilities.

B
An improving trend in ESG capabilities.

C
A static trend in ESG capabilities.

D
An volatile trend in ESG capabilities.
You answered : B - An improving trend in ESG capabilities.
The correct answer is: B - An improving trend in ESG capabilities.
Explanation
LGT Capital Partners has tracked the development of ESG capabilities among its managers for seven years, with the data
illustrating the progress among the hedge fund managers it monitors.
Reference: Chapter 8 section 3

Question 48
[4030412] Which of the following fixed income strategies demonstrates the lowest level of ESG integration?
A
Buy and maintain.

B
Government.

C
Unconstrained.

D
High yield.
You answered : A - Buy and maintain.
The correct answer is: B - Government.
Explanation
Government debt currently demonstrates the lowest fixed income ESG ratings.
Reference: Chapter 8 section 4

Question 49
[4030413]

Which of the following are considered in the World Bank’s World Governance Indicators:

i. Government effectiveness.
ii. Political stability.

iii. Corruption levels.

iv. Rule of law

A
i, ii and iii only.

B
i, iii and iv only.

C
ii, iii and iv only.

D
i, ii, iii and iv.
You answered : C - ii, iii and iv only.
The correct answer is: D - i, ii, iii and iv.
Explanation
ESG integration approaches that lend themselves well to equities and corporate debt run into a number of difficulties when
applied to sovereign debt. Fortunately, investors benefit from a growing pool of sovereign investment research resources. Not
surprisingly, many of these resources focus on governance. Many ESG-focused sovereign debt investors begin by building and
integrating an ESG framework based on the World Bank’s World Governance Indicators which consider: A country’s
governance score; Its rankings on political stability, voice and accountability, government effectiveness, rule of law, regulatory
quality, control of corruption.
Reference: Chapter 8 section 4

Question 50
[4030414] Which of the following may be described as representing market risk, such as an economic recession, that cannot be
resolved through portfolio diversification, alone?
A
Variance.

B
Idiosyncratic risk.

C
Systematic risk.

D
VaR
You answered : C - Systematic risk.
The correct answer is: C - Systematic risk.
Explanation
Systematic risk may be described as representing market risk that cannot be resolved through portfolio diversification alone.
Reference: Chapter 8 section 4

Question 51
[4007661] What should you do before an analyst meeting at which you wish to engage with a company on social issues?
A
Establish clear objectives.

B
Contact the Investor Forum for advice and assistance.

C
Arrange a meeting with the company chair.

D
Make a pre-meeting call to the CEO to raise the issue.
You answered : A - Establish clear objectives.
The correct answer is: A - Establish clear objectives.
Explanation
The first stage in the engagement process is for Investors to define the scope of the engagement and prioritise their
engagement activities carefully in order to ensure it is value-adding for their clients/beneficiaries and impactful in terms of
delivering improved corporate practices.
Reference: Chapter 6 Section 5

Question 52
[4007663] In an ESG context, when integrating ESG to manage portfolio risk what are the characteristics of tail risk?
A
Risks that are short-term in nature and describe a minor movement in the risk profile of an asset.

B
Risks that are short-term in nature and describe a major movement in the risk profile of an asset.

C
Risks that are long-term in nature and describe a minor movement in the risk profile of an asset.

D
Risks that are long-term in nature and describe a major movement in the risk profile of an asset.
You answered : D - Risks that are long-term in nature and describe a major movement in the risk profile of an asset.
The correct answer is: D - Risks that are long-term in nature and describe a major movement in the risk profile of an asset.
Explanation
In an ESG context, tail risks are generally long-term in nature and describe a significant change or move by several standard
deviations in the risk profile of an asset. Depending on the position size in a portfolio, the potential volatility of such an asset
may carry significant implications for the portfolio’s overall risk profile and to its potential risk-adjusted returns.
Reference: Chapter 8 section 4

Question 53
[4007667] How can digital disruption be best described?
A
Digital disruption is the change that occurs when new digital technologies and business models positively affect the value
proposition of existing goods but poses risks for services..

B
Digital disruption is the change that occurs when new digital technologies and business models pose risks for the value
proposition of existing goods and services.

C
Digital disruption is the change that occurs when new digital technologies and business models affect the value proposition of
existing goods and services, offering risks and opportunities.

D
Digital disruption is the change that occurs when new digital technologies and business models affect the value proposition of
existing goods and services, offering opportunities without risks.
You answered : C - Digital disruption is the change that occurs when new digital technologies and business models affect
the value proposition of existing goods and services, offering risks and opportunities.
The correct answer is: C - Digital disruption is the change that occurs when new digital technologies and business models
affect the value proposition of existing goods and services, offering risks and opportunities.
Explanation
Digital disruption is the change that occurs when new digital technologies and business models affect the value proposition of
existing goods and services, offering risks and opportunities. The trend is closely related to the increased automation and rise of
artificial intelligence (AI)..
Reference: Chapter 4 Section 1

Question 54
[4007668] ESG investing takes into account the potential material impact from a range of categorically defined risks and
opportunities, but what do the letters E, S and G stand for?
A
Environmental, social and governance.

B
Ethical, social and governance.

C
Environmental, social and government.

D
Ethical, social and government.
You answered : A - Environmental, social and governance.
The correct answer is: A - Environmental, social and governance.
Explanation
ESG stands for environmental, social and governance.
Reference: Chapter 1 Section 1

Question 55
[4007669] Which US state is home to more than half of all publicly traded corporations in the USA?
A
Washington DC.

B
Delaware.

C
New York.

D
California.
You answered : B - Delaware.
The correct answer is: B - Delaware.
Explanation
Delaware is home to more than half of all publicly traded corporations in the USA. As a result, the decisions made by the
Delaware courts are of disproportionate importance to US corporate life.
Reference: Chapter 5 Section 5

Question 56
[4007671] Which social factor does the Bangladesh Investor Initiative aim to address?
A
Employee diversity.

B
Human rights..

C
Labour rights.

D
Human capital development..
You answered : C - Labour rights.
The correct answer is: B - Human rights..
Explanation
The Bangladesh Investor initiative is an investor coalition comprising 250 institutional investors, representing over US$4.5 trillion
in assets under management that was formed in May of 2013 to urge a strong corporate response to the Rana Plaza disaster, a
health and safety issue.
Reference: Chapter 4 Section 2

Question 57
[4007673] Within ESG investment mandates, in which of the following ways should alignment between asset manager and
client be specified?
A
Alignment should be designed so that the time frames and structures of portfolio manager remuneration closely reflects the
performance experienced by clients.

B
Alignment should mean that portfolio managers respond to the clearly expressed intentions of clients and report as fully as
required.

C
Alignment should ensure engagement with companies and issuers on ESG issues and, for listed equities, voting all
shareholdings.

D
Alignment should be clearly specified within the investment mandate through more regular performance reviews.
You answered : B - Alignment should mean that portfolio managers respond to the clearly expressed intentions of clients
and report as fully as required.
The correct answer is: A - Alignment should be designed so that the time frames and structures of portfolio manager
remuneration closely reflects the performance experienced by clients.
Explanation
Alignment should be designed so that the time frames and structures of portfolio manager remuneration closely reflects the
performance experienced by clients, and accountability should mean that portfolio managers respond to the clearly expressed
intentions of clients and report as fully as required.
Reference: Chapter 9 Section 1

Question 58
[4007674] Sustainable Development Goals (SDGs) have been established by the United Nations and, by 2015, had been
adopted by 150 UN member states. How many SDGs are there?
A
There are 9 SDGs.

B
There are 14 SDGs.

C
There are 17 SDGs.

D
There are 22 SDGs.
You answered : C - There are 17 SDGs.
The correct answer is: C - There are 17 SDGs.
Explanation
There are 17 SDGs in various areas such as poverty, gender equality and life below water.
Reference: Chapter 1 Section 5

Question 59
[4007679] In addition to the UK Stewardship Code's list of escalation measures that can be considered to advance
engagements, there are a handful of additional methods used by some as part of their escalation model. Which of the following
does not fall within these additional measures?
A
Formally adding the company to an exclusion list.

B
Voting at the annual general meeting.

C
Seeking dialogue with other stakeholders.

D
Seeking governance improvements through legal methods.
You answered : B - Voting at the annual general meeting.
The correct answer is: B - Voting at the annual general meeting.
Explanation
The UK Stewardship Code sets out a list of escalation measures that can be considered to advance engagements. However
there are a handful of additional methods used by some as part of their escalation model, including: writing a formal letter
setting out concerns, usually following one of the above meetings, and typically to the chair; such letters are usually private, but
may occasionally be leaked publicly if frustrations worsen; seeking dialogue with other stakeholders, including regulators,
banks, creditors, customers, suppliers, the workforce and NGOs (stakeholder dialogue is most typically a tool in European
markets, but is increasingly being used elsewhere as well); seeking governance improvements and/or damages through legal
remedies or arbitration; formally adding the company to an exclusion list, or otherwise exiting or threatening to exit from the
investment.
Reference: Chapter 6 Section 5

Question 60
[4007686] A factor has been identified which offers a positive opportunity to a company. How may this be integrated into an
analyst’s assessment of that company?
A
By reducing forecast costs and/or reducing the cost of capital.

B
By increasing forecast costs and/or reducing the cost of capital.

C
By reducing forecast costs and/or increasing the cost of capital.

D
By increasing forecast costs and/or increasing the cost of capital.
You answered : B - By increasing forecast costs and/or reducing the cost of capital.
The correct answer is: A - By reducing forecast costs and/or reducing the cost of capital.
Explanation
When adjusting models based on ESG assessments, the analyst may: (1) Make explicit forecast profit and loss, balance sheet
and cash flows adjustments for specific factors such as litigation, stranded asset write-offs, etc.; (2) Adjust the cost of capital
used in any discounted cash flow (DCF) model for changing ESG risk levels; (3) Make valuation ratio adjustments based on
peer group comparisons.
Reference: Chapter 7 Section 3

Question 61
[4007687]

Which of the following have been proposed by the Pensions and Lifetime Savings Association (PLSA) as possible disclosures to

demonstrate the management and monitoring of ESG risks and opportunities?

i Sector level ESG analysis.

ii Stock level ESG analysis.

iii Material changes to portfolio companies’ ESG performance.

A
i and ii only.

B
i and iii only.

C
ii and iii only.

D
i, ii and iii.
You answered : D - i, ii and iii.
The correct answer is: C - ii and iii only.
Explanation
PLSA proposed possible disclosures to demonstrate the management and monitoring of ESG risks and opportunities including:
stock level ESG analysis for top risk and performance detractors/contributors in the reporting period; any material changes to
portfolio companies’ ESG performance. Examples may include where the manager’s view of ESG risk and opportunity differs
from the market/rating agencies.
Reference: Chapter 9 Section 7

Question 62
[4007688] Which of the following is a valid challenge to the integration of ESG data within investment analysis?
A
Material ESG data must be audited.

B
Material ESG data may be incomplete.

C
Material ESG data are highly consistent.
D
Material ESG data are plentiful and easy to source.
You answered : B - Material ESG data may be incomplete.
The correct answer is: B - Material ESG data may be incomplete.
Explanation
There are many hurdles and challenges for ESG integration, including disclosure and data-related challenges, such as: data
consistency; data scarcity; data incompleteness; lack of audited data. ESG factors can be judged material and useful but also
the data may be incomplete.
Reference: Chapter 7 Section 3

Question 63
[4007689] When an economy has an ageing population, which of the following statements is true?
A
Older people spend more per head on consumer goods.

B
Health care costs rise sharply when populations age.

C
The pressures on national tax revenues are reduced.

D
The ratio between the active and the inactive part of the workforce rises.
You answered : B - Health care costs rise sharply when populations age.
The correct answer is: B - Health care costs rise sharply when populations age.
Explanation
An ageing population has substantial effects on society. Firstly, the ratio between the active and the inactive part of the
workforce drops, impacting national tax revenues and challenging pension systems. Furthermore, older people have higher
accumulated savings per head than younger people, but spend less on consumer goods, which is a business risk for some
industries. In some categories, such as health care, expenditure rises sharply when populations age.
Reference: Chapter 4 Section 1

Question 64
[4007690] The chairman of the board in a typical UK company plays a key role in the management of the business for a number
of reasons. Which of the following is not one of those reasons?
A
The chair sets the agenda for board discussions.

B
The chair influences succession planning.

C
The chair ensures proper debate of strategic plans.

D
The chair is typically also the CEO.
You answered : D - The chair is typically also the CEO.
The correct answer is: D - The chair is typically also the CEO.
Explanation
The chairman of the board is vital in facilitating a balanced debate in the boardroom. Consequently, many investors prefer that
the chair is an independent non-executive director. If the chair is not independent, and especially if that individual combines the
role of chair with the role of CEO, this can lead to an excessive concentration of powers and hamper the board’s ability to:
exercise their oversight responsibilities; challenge and debate performance and strategic plans; set the agenda; influence
succession planning; debate executive remuneration.
Reference: Chapter 5 Section 1

Question 65
[4007692] Which of the following best describes the concept of the triple bottom line (TBL) in the context of corporate
accounting?
A
TBL attempts to internalise within the accounts of the company all costs that have historically been externalised.

B
TBL attempts to enhance the quality of financial reporting through improvements in auditing standards and practices and
enhancements to corporate governance practices.
C
TBL attempts to clearly communicate to investors and stakeholders the value created by companies when social and
environmental issues are considered systematically in their business operations.

D
TBL attempts to improve reporting practices and enhance the quality of corporate governance practices through more suitable
accounting standards and practices.
You answered : C - TBL attempts to clearly communicate to investors and stakeholders the value created by companies
when social and environmental issues are considered systematically in their business operations.
The correct answer is: C - TBL attempts to clearly communicate to investors and stakeholders the value created by
companies when social and environmental issues are considered systematically in their business operations.
Explanation
The triple bottom line (TBL) is an accounting framework with three parts: people (the social element), planet (the environmental
or ecological component) and profit (the financial element). TBL attempts to communicate more clearly to investors and
stakeholders the value created by companies when social and environmental issues were considered systematically in their
business operations.
Reference: Chapter 1 Section 1

Question 66
[4007694] Which of the following features are investment consultants most likely to look for when assessing the ESG
capabilities of fund managers?
A
A long-term investment horizon and high portfolio turnover.

B
A short-term investment horizon and high portfolio turnover.

C
A long-term investment horizon and low portfolio turnover.

D
A short-term investment horizon and low portfolio turnover.
You answered : C - A long-term investment horizon and low portfolio turnover.
The correct answer is: C - A long-term investment horizon and low portfolio turnover.
Explanation
Investment consultants look for a long-term investment horizon and low portfolio turnover.
Reference: Chapter 7 Section 4

Question 67
[4007699] Which of the following is not covered by the OECD Guidelines for Multinational Enterprises (MNEs)?
A
Consumer interests.

B
Government ;policies.

C
Information disclosure.

D
Combating bribery.
You answered : B - Government ;policies.
The correct answer is: B - Government ;policies.
Explanation
The OECD Guidelines for Multinational Enterprises (MNEs) provide voluntary principles and standards for responsible business
conduct in areas such as employment and industrial relations, human rights, environment, information disclosure, combating
bribery, consumer interests, science and technology, competition and taxation.
Reference: Chapter 4 Section 2

Question 68
[4007700] Four concerns are typically expressed by critics in relation to ESG integration. Which of the following is not one of
those concerns?
A
Potential over-emphasis of long-term improvements.
B
Dubious assessment criteria.

C
Quality of data.

D
Too inclusive of poor companies.
You answered : A - Potential over-emphasis of long-term improvements.
The correct answer is: A - Potential over-emphasis of long-term improvements.
Explanation
The four concerns are: Too inclusive of poor companies; Dubious assessment criteria; Quality of data; Potential lack of
emphasis of long-term improvements.
Reference: Chapter 7 Section 3

Question 69
[4007701] Which of the following types of portfolio most likely lends itself to the highest level of stewardship?
A
Fundamental active equity strategy holding a concentrated portfolio

B
UK diversified equity.

C
International diversified equity.

D
All-share index tracking.
You answered : A - Fundamental active equity strategy holding a concentrated portfolio
The correct answer is: A - Fundamental active equity strategy holding a concentrated portfolio
Explanation
Concentrated portfolios are more easily resourced from a stewardship perspective. Those firms that pride themselves on being
active stewards are genuinely concentrated and can commit significant resources to these activities. And this leads to one way
of addressing the resourcing issue: portfolio managers themselves becoming more actively involved in stewardship. This is
natural for fundamental active equity managers who hold concentrated portfolios of stocks and would typically maintain a
constant dialogue with management. Stewardship becomes challenging with larger portfolios of many stocks where managers
may not have a direct dialogue with all the companies. .
Reference: Chapter 9 Section 4

Question 70
[4007702] How do equity analysts and portfolio managers seek to integrate their qualitative ESG opinions into their investment
decision-making?
A
Through a subjective overlay on a valuation.

B
Through negative screening.

C
Through positive screening.

D
Through quantitative adjustments to financial models.
You answered : D - Through quantitative adjustments to financial models.
The correct answer is: D - Through quantitative adjustments to financial models.
Explanation
Qualitative and quantitative ESG equity analysis may each be integrated into the investment process on either a qualitative or
quantitative basis. Qualitative approaches typically assess the qualitative or quantitative company data relative to its peer group,
perhaps resulting in a relative ESG score which may be used to guide the strategic or tactical asset allocation, or ESG factor
screens or tilts. Quantitative integration typically involves one of: 1. Adjusting forecast financials for specific quantified factors
such as litigation, stranded asset write-offs, etc.; 2. Adjusting the cost of capital used in any discounted cash flow (DCF) model
for changing ESG risk levels; 3 Adjusting valuation multiples based on peer group comparisons within relative valuation models
such as P/E based valuations.
Reference: Chapter 7 Section 2
Question 71
[4007703] The Inter-governmental Panel on Climate Change (IPCC) reported in November 2018 that human activities are
estimated to have caused approximately what level of global warming above pre-industrial levels?
A
Approximately 1.0°C.

B
Approximately 1.5°C.

C
Approximately 2.0°C.

D
Approximately 2.5°C.
You answered : C - Approximately 2.0°C.
The correct answer is: A - Approximately 1.0°C.
Explanation
According to a November 2018 report by the Inter-governmental Panel on Climate Change (IPCC), human activities are
estimated to have caused approximately 1.0°C of global warming above pre-industrial levels.
Reference: Chapter 3 Section 1

Question 72
[4007704] Within the sphere of corporate governance, what is the connection between agency, alignment and executive
remuneration?
A
Executive pay structures should align the interests of management with those of other stakeholders.

B
Executive pay structures should align the interests of management with those of other board members.

C
Executive remuneration should be aligned to the executive's role as agent for the board.

D
Executive remuneration should be aligned to the executive's role as agent for the shareholders.
You answered : D - Executive remuneration should be aligned to the executive's role as agent for the shareholders.
The correct answer is: D - Executive remuneration should be aligned to the executive's role as agent for the shareholders.
Explanation
Corporate governance comes down to the two ‘A’s of accountability and alignment. Since the executives of a company act as
agents for the owners, executive pay structures should align the interests of management with those of the owners, usually by
creating a balanced compensation package that includes performance-related remuneration based on long-term goals that vest
over a long-term period.
Reference: Chapter 5 Section 1

Question 73
[4007706] A company has taken significant positive steps with respect to climate change impacts, resulting in an analyst
assigning an improved ESG score, however, the analyst remains concerned about the long-term outlook for the sector in which
the company operates and the potential that these changes may have for creating stranded assets. How should this be reflected
in the analyst's valuation of the company?
A
The analyst should adjust the financial model to reflect a higher cost of capital.

B
The analyst should adjust the financial model in later years to reflect potential stranded assets write-offs.

C
There is no financial adjustment necessary since the analyst's concerns have not yet materialised.

D
The analyst should adjust the financial model to reflect potential stranded assets write-offs and apply a higher cost of capital to
reflect continuing sector-wide climate risk concerns..
You answered : D - The analyst should adjust the financial model to reflect potential stranded assets write-offs and apply a
higher cost of capital to reflect continuing sector-wide climate risk concerns..
The correct answer is: D - The analyst should adjust the financial model to reflect potential stranded assets write-offs and
apply a higher cost of capital to reflect continuing sector-wide climate risk concerns..
Explanation
If a company’s environmental management processes and policies are judged strong/weak, the cost of capital used to discount
cash flows in a DCF analysis is adjusted down/up to account for this. In addition, specific factors such as potential stranded
assets write-offs should be individually reflected in the model.
Reference: Chapter 7 Section 3

Question 74
[4007708] Which of the following is not one of the various methods by which managers may integrate ESG?
A
As a threshold requirement before investment can be considered.

B
As a factor that informs the valuation.

C
As a risk assessment that offers a level of confidence in the valuation.

D
As a performance reporting tool.
You answered : D - As a performance reporting tool.
The correct answer is: D - As a performance reporting tool.
Explanation
Different managers will integrate ESG in different ways – as a threshold requirement before investment can be considered, as a
factor that informs the valuation, as a risk assessment that offers a level of confidence in the valuation, or as a basis for
stewardship engagement, or very often a combination of two or more of these methods.
Reference: Chapter 9 Section 3

Question 75
[4007709] Single tier boards are typically most common in which of the following countries or regions?
A
Netherlands.

B
Scandinavia.

C
Japan.

D
China.
You answered : B - Scandinavia.
The correct answer is: C - Japan.
Explanation
A two-tier board structure seen in Germany, the Netherlands, Scandinavia and (at least formally) China. Single-tier (also called
unitary) boards are more typical of the UK, the USA, Japan, France and most of the rest of the world.
Reference: Chapter 5 Section 5

Question 76
[4007715] Although institutional investors tend to dominate the financial market, interest by retail investors in responsible
investing has been steadily growing. Under data compiled by the GSIA, in 2018, the retail portion had grown by what
percentage?
A
Interest by retail investors in responsible investing has grown by 15%

B
Interest by retail investors in responsible investing has grown by 25%

C
Interest by retail investors in responsible investing has grown by 35%

D
Interest by retail investors in responsible investing has grown by 45%
You answered : B - Interest by retail investors in responsible investing has grown by 25%
The correct answer is: B - Interest by retail investors in responsible investing has grown by 25%
Explanation
In 2018, the retail portion had grown to one quarter.
Reference: Chapter 2 Section 2

Question 77
[4007716] From a materiality context, how would you compare the concern that a typical US company has regarding a policy on
workers unions to that of a typical German company?
A
US companies are more concerned about having a policy on worker unions than German companies.

B
US companies are less concerned about having a policy on workers unions than German companies.

C
US companies have the same concern about having a policy on workers unions as German companies.

D
US companies have no concerns about workers unions, whilst German companies do.
You answered : B - US companies are less concerned about having a policy on workers unions than German companies.
The correct answer is: B - US companies are less concerned about having a policy on workers unions than German
companies.
Explanation
US companies are less concerned about having a policy on workers unions than German companies.
Reference: Chapter 7 Section 3

Question 78
[4007718] When trying to identify and measure a company’s environmental impact, investors would need to consider the rate of
a company’s utilisation or consumption of all but which of the following?
A
Waste.

B
Wind power.

C
Energy.

D
Water.
You answered : B - Wind power.
The correct answer is: B - Wind power.
Explanation
Investors looking to identify and measure a company’s environmental impact would need to consider the rate of a company’s
utilisation or consumption in energy, water and waste. .
Reference: Chapter 3 Section 4

Question 79
[4007719]

Systematic approaches to investment aim to determine how ESG factors impact on financial performance over time and weight

those factors accordingly. Which of the following objectives may this approach seek to satisfy?

i Increase returns.

ii Increase risks.

iii Increase risk-adjusted returns.

A
i and ii only.
B
i and iii only.

C
ii and iii only.

D
i, ii and iii.
You answered : B - i and iii only.
The correct answer is: B - i and iii only.
Explanation
Systematic investment approaches try to understand how ESG factors impact on financial performance over time and weight
those ESG factors appropriately with the aim of increasing returns, reducing risks, or improving risk-adjusted returns.
Reference: Chapter 7 Section 2

Question 80
[4007720] In respect of the issue of governance in private equity investments compared to the issue in public equity
investments, which of the following statements is true?
A
Governance is a greater issue in private equity investments.

B
Governance is not an issue in private equity investments.

C
Governance is just as much an issue in private equity investments.

D
It is not possible to draw any conclusion regarding governance in private equity investments.
You answered : A - Governance is a greater issue in private equity investments.
The correct answer is: C - Governance is just as much an issue in private equity investments.
Explanation
Governance is just as much an issue in private equity investments, although some say it is less of an issue because investors
are directly represented on the board. .
Reference: Chapter 5 Section 7

Question 81
[4007721] In respect of materiality mapping, which of the following statements is false?
A
Mapping can be done for material opportunities as well as risks.

B
Mapping data can be given as a quantitative score.

C
Mapping data can be provided in qualitative terms.

D
Mapping consistency is ensured by the Sustainability Accounting Standards Board (SASB).
You answered : D - Mapping consistency is ensured by the Sustainability Accounting Standards Board (SASB).
The correct answer is: D - Mapping consistency is ensured by the Sustainability Accounting Standards Board (SASB).
Explanation
All ESG materiality maps are helpful in providing some guidance, but investment professionals often develop their own view on
what is most material; there may be no consistency..
Reference: Chapter 7 Section 3

Question 82
[4007722] In describing ESG analysis, which of the following statements is true?
A
ESG analysis can only be quantitative, not qualitative.

B
ESG analysis can be either qualitative or quantitative.
C
ESG analysis is neither qualitative nor quantitative.

D
ESG analysis can only be qualitative, not quantitative.
You answered : B - ESG analysis can be either qualitative or quantitative.
The correct answer is: B - ESG analysis can be either qualitative or quantitative.
Explanation
ESG analysis can be either qualitative or quantitative. .
Reference: Chapter 7 Section 2

Question 83
[4007723] Globalisation offers both positive and negative impacts. Which of the following may be identified as a positive
impact?
A
Improved social well-being.

B
Reduced dependency on foreign suppliers..

C
Reduced transportation costs and resulting emissions.

D
Increased efficiency and lower costs.
You answered : D - Increased efficiency and lower costs.
The correct answer is: D - Increased efficiency and lower costs.
Explanation
Globalisation leads to increased efficiency in the markets, resulting in wider availability of products at lower costs.
Reference: Chapter 4 Section 1

Question 84
[4007724]

Elements of ESG analysis include which of the following?

i Red flag indicators.

ii Adjusting forecast financials.

iii Company questionnaires and management interviews.

iv Checks with outside experts.

A
i, ii and iv only.

B
i, iii and iv only.

C
ii, iii and iv only.

D
i, ii, iii and iv.
You answered : B - i, iii and iv only.
The correct answer is: B - i, iii and iv only.
Explanation
Elements of ESG analysis include: Red flag indicators, company questionnaires and management interviews, checks with
outside experts, watch lists, internal ESG research, external ESG research, ESG agenda items on investment committee or
Chief Information Officer (CIO) meetings.
Reference: Chapter 7 Section 2

Question 85
[4007725] Which of the following correctly describes the primary way in which investors assess the engagement performance of
managers?
A
Changes made by companies as engagement progresses, expressed as either milestones or objectives.

B
Changes made by companies as engagement progresses, expressed as ESG scores.

C
Changes made by companies as engagement progresses, expressed in terms of risk-adjusted performance.

D
Changes made by companies as engagement progresses, expressed in financial terms.
You answered : A - Changes made by companies as engagement progresses, expressed as either milestones or objectives.
The correct answer is: A - Changes made by companies as engagement progresses, expressed as either milestones or
objectives.
Explanation
Performance measurements that investors place around engagement have mostly been developed by the stewardship overlay
providers. These are schemes to reflect the changes made by companies as engagement progresses, expressed as either
milestones or objectives. The firms measure progress towards concrete change or better practice over the three or more years
of the engagement.
Reference: Chapter 9 Section 4

Question 86
[4007726]

Which of the following may be characterised as the internalisation of an externality?

i. Pollution.

ii. Excessive executive pay.

iii. Levies on landfill dumping.

Iv. Carbon taxes.

A
i and ii only.

B
iii and iv only.

C
i, ii and iv only.

D
i, ii, iii and iv.
You answered : B - iii and iv only.
The correct answer is: B - iii and iv only.
Explanation
Externalities are costs that a business does not bear itself (internalise) but instead imposes on others (externalises). A carbon
tax and levies on landfill dumping are direct costs suffered (internalised) by a business by defining a rate payable in relation to
greenhouse gas emissions or waste management (an externality), i.e. an externality that has been internalised.
Reference: Chapter 1 Section 3

Question 87
[4007727] An investment has an E score of 55.9 within a portfolio with an E score of 81.7. The portfolio benchmark has an E
score of 84.1 and the market has an E score of 83.6. What is the investment's detraction level?
A
The detraction level is 112.3.
B
The detraction level is 84.1.

C
The detraction level is 28.2.

D
The detraction level is 0.
You answered : C - The detraction level is 28.2.
The correct answer is: C - The detraction level is 28.2.
Explanation
The detraction level is the amount by which the company's ESG score falls below that of the portfolio's benchmark score. Given
the portfolio's benchmark score is 84.1 and the company's ESG score differs (hence the detraction level cannot be zero), the
only viable option is 28.2. If we wished to calculate the figure we would get 28.2 (84.1 – 55.9)..
Reference: Chapter 9 Section 6

Question 88
[4007728] Which type of investing would include seeking improving access to clean water or to energy services?
A
Best-in-class investing.

B
Qualitative investing.

C
Thematic investing.

D
Negative screening.
You answered : C - Thematic investing.
The correct answer is: C - Thematic investing.
Explanation
Investment ideas can be generated by themes associated with specific ESG megatrends, such as seeking improving access to
clean water or to energy services; this is commonly referred to as thematic investing. .
Reference: Chapter 7 Section 3

Question 89
[4007729] In making their business decisions, there is a list of business stakeholders that board directors must consider. Which
of the following should the board exclude from this list?
A
Local communities.

B
Senior management.

C
Suppliers.

D
Customers.
You answered : B - Senior management.
The correct answer is: B - Senior management.
Explanation
The role of directors and other senior managers is to run the companies they work for on behalf of the owners. They must act
for the owners, not themselves, and ensure the long-term success of the business by, amongst other things, considering ESG
issues and the impacts on external stakeholders such as customers, suppliers, local communities, etc.
Reference: Chapter 5 Section 1

Question 90
[4007734] Globally, how many people are currently estimated to be in forced labour?
A
5 million.
B
10 million.

C
20 million.

D
25 million.
You answered : C - 20 million.
The correct answer is: D - 25 million.
Explanation
Globally, 25 million people are estimated to be in forced labour.
Reference: Chapter 4 Section 2

Question 91
[4007737] Which of the following will the Investor Forum not achieve through engagement?
A
Appropriate risk management by companies.

B
Changed company behaviours.

C
Efficient capital allocation by investors.

D
Delivery of corporate purpose and culture.
You answered : C - Efficient capital allocation by investors.
The correct answer is: C - Efficient capital allocation by investors.
Explanation
Through engagement, the Investor Forum hope to achieve: Changed company behaviours; Efficient capital allocation by
companies; Appropriate risk management by companies; Preserved/enhanced value; Delivery of corporate purpose and culture,
through effective oversight.
Reference: Chapter 6 Section 2

Question 92
[4007738] How would a purely qualitative equity investor incorporate ESG risk factors into their investment decision framework?
A
Qualitative adjustment to relative valuation models.

B
Qualitative adjustment to forecast cash flows.

C
Qualitative adjustment to the cost of capital.

D
Qualitative adjustment at the end of the analysis process
You answered : D - Qualitative adjustment at the end of the analysis process
The correct answer is: D - Qualitative adjustment at the end of the analysis process
Explanation
ESG factors may be integrated in either: a purely qualitative manner, e.g. opinion on quality of management added to the
investment thesis (the reasoned argument for a particular investment strategy); or in a quantitative manner, e.g. impact on
forecast profits, valuation models/valuation ratios, etc. Some techniques could be considered a hybrid of both techniques, for
example, scorecards, where a qualitative judgment is turned into a quantitative score.
Reference: Chapter 7 Section 2

Question 93
[4007740] Consumer protection in respect of purchased products is typically safeguarded by all but one of the following. Which
one is the exception?
A
By enforcing product safety.
B
By enforcing health and safety.

C
By distributing consumer-related information.

D
By preventing deceptive marketing.
You answered : C - By distributing consumer-related information.
The correct answer is: B - By enforcing health and safety.
Explanation
Consumer protection is based on consumer rights, or the idea that consumers have an inherent right to basic health and safety.
These are safeguarded by: enforcing product safety; distributing consumer-related information; preventing deceptive marketing.
Reference: Chapter 4 Section 2

Question 94
[4007742] In relation to ESG mapping and the examination of a portfolio's ESG risks, which of the following statements is true?
A
ESG mapping is based on a quantifiable historical assessment.

B
ESG mapping is a precise science that shows the exact ESG risks faced.

C
ESG mapping identifies specific key E, S, and G exposures.

D
ESG mapping shows the unequivocal ESG risk irrespective of the analyst concerned.
You answered : C - ESG mapping identifies specific key E, S, and G exposures.
The correct answer is: C - ESG mapping identifies specific key E, S, and G exposures.
Explanation
EGS mapping is a technique for determining which ESG issues may be most material to companies in a sector. We should
note, however, that mapping is not an exact science, and there might be important differences between what one investor
considers most material when compared to what other investors consider most material, even when analysing the same
company. This is because it is typically a forecast of judgement on how much one ESG or risk factor will impact a financial
metric such as future cash flow.
Reference: Chapter 7 Section 3

Question 95
[4007744] The UK was the first country to develop a stewardship code which is now part of the UK law. Which body is
responsible for enforcing the Stewardship Code?
A
The Financial Reporting Council (FRC).

B
The Investor Forum.

C
The Financial Conduct Authority (FCA).

D
The Pension and Lifetime Savings Association (PSLA).
You answered : C - The Financial Conduct Authority (FCA).
The correct answer is: A - The Financial Reporting Council (FRC).
Explanation
Within the UK, the body responsible for releasing and enforcing the Stewardship Code is the Financial Reporting Council (FRC).
Reference: Chapter 6 Section 3

Question 96
[4007752] With regard to the engagement activities of investors, which of the following statements is most accurate?
A
Passive investors typically start with the company and specific business issue and develop a tailored engagement approach.
B
Passive investors typically start with the company and specific business issue and develop a generic engagement approach for
all investee companies.

C
Active investors typically start with the company and specific business issue and develop a tailored engagement approach.

D
Active investors typically start with the company and specific business issue and develop a generic engagement approach for all
investee companies.
You answered : C - Active investors typically start with the company and specific business issue and develop a tailored
engagement approach.
The correct answer is: C - Active investors typically start with the company and specific business issue and develop a
tailored engagement approach.
Explanation
Active investors typically start with the company and specific business issue and develop a tailored engagement approach.
Reference: Chapter 6 Section 4

Question 97
[4007753] Which of the following is a growing investor initiative that focuses and engages on the risks and opportunities linked
to intensive livestock production?
A
FAIRR.

B
PLWF.

C
FTCFD.

D
FPIC.
You answered : A - FAIRR.
The correct answer is: A - FAIRR.
Explanation
A growing investor initiative, which is focused and engaged on the risks and opportunities linked to intensive livestock
production is FAIRR (Farm Animal Investment Risk and Return).
Reference: Chapter 4 Section 2

Question 98
[4007757] The thematic approaches to the integration of ESG analysis into investment decisions are best described in which of
the following ways?
A
Thematic funds typically integrate ESG factors alongside other factors, such as value, size, momentum, growth and volatility.

B
Thematic funds may assess alignment with priority themes such as those of ESG factors they wish to prioritise.

C
Thematic funds use ESG data to adjust company or sector weights after parsing the ESG data through rules-based formulae.

D
Thematic funds typically gather data that is similar to fundamental investors but tends to be over larger datasets.
You answered : B - Thematic funds may assess alignment with priority themes such as those of ESG factors they wish to
prioritise.
The correct answer is: B - Thematic funds may assess alignment with priority themes such as those of ESG factors they
wish to prioritise.
Explanation
Thematic funds may assess alignment with ESG factors they wish to prioritise.
Reference: Chapter 7 Section 2

Question 99
[4007759] How would you best describe the role of the auditor in relation to the financial statements and to the narrative
elements of the annual report and accounts?
A
The auditor provides assurance on the financial statements but has no responsibility with respect to the narrative reports.

B
The auditor provides assurance on the financial statements and reports any inconsistencies in the narrative reports.

C
The auditor provides assurance on the financial statements and on any financial figures contained in the narrative reports.

D
The auditor provides assurance on both the financial statements and the narrative reports.
You answered : C - The auditor provides assurance on the financial statements and on any financial figures contained in the
narrative reports.
The correct answer is: B - The auditor provides assurance on the financial statements and reports any inconsistencies in the
narrative reports.
Explanation
The auditor is there to provide an independent pair of eyes assessing the financial reports prepared by management, and to
provide some assurance that those reports fairly represent the performance and position of the business. There is no absolute
assurance that the numbers are correct, nor is there certainty that there is no fraud within the business. The auditor checks and
assures the financial statements in detail; their role in relation to the words and numbers in the more discursive front half of the
annual report is less stringent. The auditor must read this segment and should comment if they discover something that is
inconsistent with what they have learned through the process of the audit.
Reference: Chapter 5 Section 6

Question 100
[4007760] An investment approach that seeks to select or use assets that minimise natural and social resource depletion best
describes which of the following?
A
Financial investing.

B
Responsible investing.

C
Impact investing.

D
Sustainable investing.
You answered : D - Sustainable investing.
The correct answer is: D - Sustainable investing.
Explanation
Sustainable investing would best be described as an investment approach that seeks to select or use assets that minimise
natural and social resource depletion.
Reference: Chapter 1 Section 2

You might also like